Why would a society of witches choose to have more boys than girls? [closed]

The name of the pictureThe name of the pictureThe name of the pictureClash Royale CLAN TAG#URR8PPP











up vote
6
down vote

favorite
3












In my story's world, witchcraft is a respected institution, with the most powerful practitioners being at the top echelons of society. Due to this, society traces its lineage through matrilineal lines. Witchcraft is exclusive to the female line. Ritual magic involves using the mana of the world and directing it in a way to suit you (altering terrain, changing the weather, erecting barriers, etc). It requires chanting, numerous ingredients, and a group of people depending on the spell. Society is divided up into covens, with the most powerful witches being top leaders in their covern. Ritual magic is essential to society, so logically the more witches a coven has, the more powerful and influential it would be. However, this matriarchal system has more males than females in these covens.



There is a spell that allows one to choose the sex of their offspring. This is a simple spell requiring only the individual , some incantations, and a number of ingredients. Why would this civilization choose to have more boys than girls If they lack access to magic? What would be the benefit?







share|improve this question














closed as primarily opinion-based by kingledion, Cadence, Alexander von Wernherr, James♦ Aug 31 at 19:04


Many good questions generate some degree of opinion based on expert experience, but answers to this question will tend to be almost entirely based on opinions, rather than facts, references, or specific expertise. If this question can be reworded to fit the rules in the help center, please edit the question.










  • 7




    Also, this has a good chance of being close for being Primarily Opinion Based.
    – RonJohn
    Aug 29 at 18:52






  • 4




    There is no real criteria that makes one answer better than another, so I think this question is primarily opinion-based.
    – kingledion
    Aug 29 at 21:19






  • 2




    @abarnert you should delete your comment and make a full answer out of it instead.
    – Ister
    Aug 30 at 8:37






  • 2




    because they taste better?
    – PlasmaHH
    Aug 30 at 8:46






  • 1




    @immibis there's usually some non-fiction element that you can apply to make an answer. This however is fiction from start to end, so it can be pretty much anything. "Because there's only 5 quarts of magic in the world and they don't want to share" is as good an answer as "because they feed on human flesh since it's more salty"... because there's no possible non-fiction reasoning that can differentiate the two.
    – UKMonkey
    Aug 30 at 11:59















up vote
6
down vote

favorite
3












In my story's world, witchcraft is a respected institution, with the most powerful practitioners being at the top echelons of society. Due to this, society traces its lineage through matrilineal lines. Witchcraft is exclusive to the female line. Ritual magic involves using the mana of the world and directing it in a way to suit you (altering terrain, changing the weather, erecting barriers, etc). It requires chanting, numerous ingredients, and a group of people depending on the spell. Society is divided up into covens, with the most powerful witches being top leaders in their covern. Ritual magic is essential to society, so logically the more witches a coven has, the more powerful and influential it would be. However, this matriarchal system has more males than females in these covens.



There is a spell that allows one to choose the sex of their offspring. This is a simple spell requiring only the individual , some incantations, and a number of ingredients. Why would this civilization choose to have more boys than girls If they lack access to magic? What would be the benefit?







share|improve this question














closed as primarily opinion-based by kingledion, Cadence, Alexander von Wernherr, James♦ Aug 31 at 19:04


Many good questions generate some degree of opinion based on expert experience, but answers to this question will tend to be almost entirely based on opinions, rather than facts, references, or specific expertise. If this question can be reworded to fit the rules in the help center, please edit the question.










  • 7




    Also, this has a good chance of being close for being Primarily Opinion Based.
    – RonJohn
    Aug 29 at 18:52






  • 4




    There is no real criteria that makes one answer better than another, so I think this question is primarily opinion-based.
    – kingledion
    Aug 29 at 21:19






  • 2




    @abarnert you should delete your comment and make a full answer out of it instead.
    – Ister
    Aug 30 at 8:37






  • 2




    because they taste better?
    – PlasmaHH
    Aug 30 at 8:46






  • 1




    @immibis there's usually some non-fiction element that you can apply to make an answer. This however is fiction from start to end, so it can be pretty much anything. "Because there's only 5 quarts of magic in the world and they don't want to share" is as good an answer as "because they feed on human flesh since it's more salty"... because there's no possible non-fiction reasoning that can differentiate the two.
    – UKMonkey
    Aug 30 at 11:59













up vote
6
down vote

favorite
3









up vote
6
down vote

favorite
3






3





In my story's world, witchcraft is a respected institution, with the most powerful practitioners being at the top echelons of society. Due to this, society traces its lineage through matrilineal lines. Witchcraft is exclusive to the female line. Ritual magic involves using the mana of the world and directing it in a way to suit you (altering terrain, changing the weather, erecting barriers, etc). It requires chanting, numerous ingredients, and a group of people depending on the spell. Society is divided up into covens, with the most powerful witches being top leaders in their covern. Ritual magic is essential to society, so logically the more witches a coven has, the more powerful and influential it would be. However, this matriarchal system has more males than females in these covens.



There is a spell that allows one to choose the sex of their offspring. This is a simple spell requiring only the individual , some incantations, and a number of ingredients. Why would this civilization choose to have more boys than girls If they lack access to magic? What would be the benefit?







share|improve this question














In my story's world, witchcraft is a respected institution, with the most powerful practitioners being at the top echelons of society. Due to this, society traces its lineage through matrilineal lines. Witchcraft is exclusive to the female line. Ritual magic involves using the mana of the world and directing it in a way to suit you (altering terrain, changing the weather, erecting barriers, etc). It requires chanting, numerous ingredients, and a group of people depending on the spell. Society is divided up into covens, with the most powerful witches being top leaders in their covern. Ritual magic is essential to society, so logically the more witches a coven has, the more powerful and influential it would be. However, this matriarchal system has more males than females in these covens.



There is a spell that allows one to choose the sex of their offspring. This is a simple spell requiring only the individual , some incantations, and a number of ingredients. Why would this civilization choose to have more boys than girls If they lack access to magic? What would be the benefit?









share|improve this question













share|improve this question




share|improve this question








edited Aug 29 at 19:03

























asked Aug 29 at 18:47









Incognito

1,62221624




1,62221624




closed as primarily opinion-based by kingledion, Cadence, Alexander von Wernherr, James♦ Aug 31 at 19:04


Many good questions generate some degree of opinion based on expert experience, but answers to this question will tend to be almost entirely based on opinions, rather than facts, references, or specific expertise. If this question can be reworded to fit the rules in the help center, please edit the question.






closed as primarily opinion-based by kingledion, Cadence, Alexander von Wernherr, James♦ Aug 31 at 19:04


Many good questions generate some degree of opinion based on expert experience, but answers to this question will tend to be almost entirely based on opinions, rather than facts, references, or specific expertise. If this question can be reworded to fit the rules in the help center, please edit the question.









  • 7




    Also, this has a good chance of being close for being Primarily Opinion Based.
    – RonJohn
    Aug 29 at 18:52






  • 4




    There is no real criteria that makes one answer better than another, so I think this question is primarily opinion-based.
    – kingledion
    Aug 29 at 21:19






  • 2




    @abarnert you should delete your comment and make a full answer out of it instead.
    – Ister
    Aug 30 at 8:37






  • 2




    because they taste better?
    – PlasmaHH
    Aug 30 at 8:46






  • 1




    @immibis there's usually some non-fiction element that you can apply to make an answer. This however is fiction from start to end, so it can be pretty much anything. "Because there's only 5 quarts of magic in the world and they don't want to share" is as good an answer as "because they feed on human flesh since it's more salty"... because there's no possible non-fiction reasoning that can differentiate the two.
    – UKMonkey
    Aug 30 at 11:59













  • 7




    Also, this has a good chance of being close for being Primarily Opinion Based.
    – RonJohn
    Aug 29 at 18:52






  • 4




    There is no real criteria that makes one answer better than another, so I think this question is primarily opinion-based.
    – kingledion
    Aug 29 at 21:19






  • 2




    @abarnert you should delete your comment and make a full answer out of it instead.
    – Ister
    Aug 30 at 8:37






  • 2




    because they taste better?
    – PlasmaHH
    Aug 30 at 8:46






  • 1




    @immibis there's usually some non-fiction element that you can apply to make an answer. This however is fiction from start to end, so it can be pretty much anything. "Because there's only 5 quarts of magic in the world and they don't want to share" is as good an answer as "because they feed on human flesh since it's more salty"... because there's no possible non-fiction reasoning that can differentiate the two.
    – UKMonkey
    Aug 30 at 11:59








7




7




Also, this has a good chance of being close for being Primarily Opinion Based.
– RonJohn
Aug 29 at 18:52




Also, this has a good chance of being close for being Primarily Opinion Based.
– RonJohn
Aug 29 at 18:52




4




4




There is no real criteria that makes one answer better than another, so I think this question is primarily opinion-based.
– kingledion
Aug 29 at 21:19




There is no real criteria that makes one answer better than another, so I think this question is primarily opinion-based.
– kingledion
Aug 29 at 21:19




2




2




@abarnert you should delete your comment and make a full answer out of it instead.
– Ister
Aug 30 at 8:37




@abarnert you should delete your comment and make a full answer out of it instead.
– Ister
Aug 30 at 8:37




2




2




because they taste better?
– PlasmaHH
Aug 30 at 8:46




because they taste better?
– PlasmaHH
Aug 30 at 8:46




1




1




@immibis there's usually some non-fiction element that you can apply to make an answer. This however is fiction from start to end, so it can be pretty much anything. "Because there's only 5 quarts of magic in the world and they don't want to share" is as good an answer as "because they feed on human flesh since it's more salty"... because there's no possible non-fiction reasoning that can differentiate the two.
– UKMonkey
Aug 30 at 11:59





@immibis there's usually some non-fiction element that you can apply to make an answer. This however is fiction from start to end, so it can be pretty much anything. "Because there's only 5 quarts of magic in the world and they don't want to share" is as good an answer as "because they feed on human flesh since it's more salty"... because there's no possible non-fiction reasoning that can differentiate the two.
– UKMonkey
Aug 30 at 11:59











23 Answers
23






active

oldest

votes

















up vote
41
down vote













Males can be used as unskilled labor, and you need more unskilled than skilled workers (at least with medieval technology). Or males are used as cannon fodder in wars, or as internal police force, which increases their mortality.



More female offspring:



  • Require more teaching effort from mother. B/c untamed magical powers can be very destructive. Magical powers are inherited, and mother is responsible for destruction caused by her child (if she survives, ofc).

  • Will fight more between themselves, especially is magical power is limited or shared within a family.

  • Are more likely to rebel against the mother. More children mean weaker bond between mother and each child, and higher chance than any one child starts feeling that mother favors other child more than them.

Finally, witchcraft will likely extend life, but only for the witch herself. So to keep gender the ratio constant, you have to have fewer female newborns.



PS For modern science, it is a lot easier to pick gender of the child on conception than to change gender of a growing fetus. If the witch can do same things with her magic, she might as well choose to clone herself rather than mix genes with some male.






share|improve this answer


















  • 9




    That also feels like a very good argument to make daughters 'daughters of their mother' and sons 'sons of society'. Basically a society could have a lot of structures to teach boys in bulk for their unskilled labor, while the girls are kept with their mother for proper training
    – Patrice
    Aug 29 at 22:45






  • 1




    +1 for your remark about different lifespans, this seems like a very good argument!
    – Bananenaffe
    Aug 30 at 6:00










  • @Patrice In this case woman will not have any incentive to have boys. It will be overly complicated and painful way to make abortion.
    – talex
    Aug 30 at 7:37










  • -1: lots of assumptions and most of them I disagree with. Men already have a higher mortality rate without magic - and make up less than 50% of the population because it quite frankly, doesn't help the species survive. One man can reproduce with many women just as effectively as one. This won't be "magically" offset by unskilled labour (you don't need it, just use magic); policing/fighting (how can a non-magic user combat a magic user?). And the idea that magic users would extend life is an assumption challenged in many stories.
    – UKMonkey
    Aug 30 at 11:55










  • This makes sence except that there would be a lot more girls than boys. I think restricting the birth rate/percentage of girls could be quite a cool idea. For example because the mother is a magic user and the girl would inherit the megic a lot of girls born dead (some magic anomaly) while boys seems to be more resistant for this etc.
    – Lakatos Gyula
    Aug 30 at 12:15

















up vote
18
down vote













Witch offspring sap mana from their mothers.



You can have a witch baby but when she emerges she takes some of mother's magic with her. If you want to be at the top of your magic game you do not want to share your magic with your offspring. The uberwitches might have only boys - but then their lineage dies. Middle powered witches might opt to share with their daughters because the lineage benefit outweighs the power loss.






share|improve this answer


















  • 3




    I like this answer, particularly since it dovetails neatly with the Shakespearian/Discworld paradigm of "Maiden, Mother and Crone"
    – Ruadhan
    Aug 30 at 12:35










  • @Ruadhan What does that mean?
    – Incognito
    Aug 30 at 16:51






  • 2




    Traditionally there are three witches (eg: Macbeth),which in the Discworld books represent three major age-groups, the young maiden, the middle-aged mother and the old-aged "crone" So in WillK's scenario, the community would naturally split into that pattern because choosing to be a parent would have a direct impact on magical power. You'd naturally in a given group have more powerful adult witches with no heirs, middle-powered Mother witches and younger Maidens growing into their power, likely the child of the Mother. Honestly I think I'm talking out of my arse at this point though :P
    – Ruadhan
    Aug 31 at 8:22

















up vote
11
down vote













Inheritance is matrilineal. Having multiple daughters means that there is some controversy over who inherits. And even without controversy, there is complexity. Primogeniture might give the bulk to the firstborn, but how to provide for second-born and later daughters?



Sons are outside inheritance. Have as many as you want. Their purpose is to make good marriages or to support their sister. Perhaps the occasional dark magic ritual sacrifice to increase power. Sons increase the family power with less concern about splitting it.



One daughter makes it clear who the heir is. A second muddies those waters. And can become an aunt who might claim precedence over a niece.






share|improve this answer



























    up vote
    7
    down vote













    The bodies of men collect and concentrate the manna of the world (making more potent spells possible), but they have no ability to control or direct it. A woman will therefore become more powerful herself if she has a few sons before having daughters as she will be able to use their concentrated manna to power her spells.



    Please note that a civilization can't make choices, only the individuals in the civilization can, so the choice has to make sense in a per person context.






    share|improve this answer



























      up vote
      5
      down vote













      Magic is sex-based, so the power is based on the amount of sex. A woman can have more sex than a man can, so having more men increases the total power.






      share|improve this answer
















      • 1




        Why does the sex have to be with a witch-born male?
        – RonJohn
        Aug 29 at 21:24










      • On what basis can she do t hat?
        – Azor Ahai
        Aug 29 at 23:18






      • 1




        @AzorAhai Women do not have a refractory period. Men can only have sex if they can maintain an erection, which they cannot do 100% of the time, while women can (strictly mechanically speaking) have sex whenever--24 hours a day, if they want.
        – Logan R. Kearsley
        Aug 31 at 12:19

















      up vote
      4
      down vote













      Males born of witches can't access magic, but they are immune to magic by virtue of their lineage and a quirk of how magic works.



      Thus, they're good -- and devoted -- warriors in the internecine battles between covens.






      share|improve this answer




















      • Males might be immune to magic spells themselves, but they're most likely not immune to damage from objects being moved by magic. Imagine what a bunch of rocks propelled through the air by magic, fast as the hawk flies, can do to a group of men charging at you. And since they're immune to magic, you can't cast defensive spells on them. Or just use a ritual to swallow them up into the earth.
        – Nzall
        Aug 30 at 9:37











      • @Nzall interesting points. Their witch mothers & sisters could create magic shields in front and under them.
        – RonJohn
        Aug 30 at 12:29










      • I was just trying to clarify how being immune to magic does not mean you can't be harmed by a magic user. There are loads of ways for magic to affect a world in such a way that it becomes lethal even to people immune to magic. In the Avatar Cartoon, an Airbender once sucked the air out of a room, killing everyone in it. In Codex Alera, a group of Earthcrafters pulled all the oil and coal to right below the surface which was then ignited by Firecrafters. Shortly after, a watercrafter diverted a river to that field. Magic immunity can be both useful and dangerous.
        – Nzall
        Aug 30 at 13:03










      • @Nzall anything you can do to a magic immune, you can also do to the magic susceptible; the reverse is not true. Thus, being immune to magic is better -- in this case -- than being susceptible. It doesn't make you immortal, though, and should not make you arrogant.
        – RonJohn
        Aug 30 at 14:29

















      up vote
      4
      down vote













      Untrained witches are dangerous to everyone. And you can train only so many at once.



      Every daughter of a witch has magic. But it is training that makes a witch out of a girl. Without training, magic is still there, but it is spontaneous, wild, bursting out at random. None wants to be around an untrained witch, as you have to maintain strongest protective charms at all time and still won't be completely sure if that would be enough.



      On the other hand, the training involved is long and deeply personal. One full-fledged witch can train only one apprentice at a time. And the training up to the moment when she can be graduated and the next apprentice gets accepted takes two decades. Thus, it is extremely rare for a witch to mentor more than two girls over her lifetime (she would graduate the second one in her sixties at the earliest). And many elect to train only a single designated successor.



      Of course, basic control gets taught early on, and you can have a non-volatile girl just a year after her magic manifests. Yet, without further training she wouldn't contribute much to the coven, and it would be such a waste (especially for her mentor) to just cut training short there. And if an unclaimed girl comes of age for whatever reason (say, her designated mentor met an accident), then either someone free (and unwilling) has to teach her, or someone has to halt teaching their current apprentice to deal with a new menace.



      Thus, it's not that boys are preferred. But there is a hard limit on how many girls are acceptable, and everyone else has to be boys. Assuming witches aren't too chaste, yet don't use contraception, this will be a natural consequence.






      share|improve this answer



























        up vote
        3
        down vote














        Eugenics.



        In a medieval setup they will not have access to DNA sequencing or even knowledge about DNA but the witches know (by methodic observation and deep knowledge of nature) how the genetic inheritance work.



        Sperm have more variety in genetic recombination than egg cells (spermatocytogenesis result in four cells while oogenesis produces only one) so the genetic content from sperm is valuable for eugenic process.



        So, witches are told by their coven leader to give birth to males and with whom to mate, only chosen witches are allowed to give birth to females.




        In the end, all of us know where this setup leads to...






        share|improve this answer



























          up vote
          2
          down vote













          Out of kindness



          Because there is somehow a great psychic/psychological burden to witchcraft that it is, in itself, some kind of great curse or scouring: the greatest curse the witch can craft being to induce witchcraft in another. This can be the case despite external benefits. There are many stories where supernatural gifts are a burden to the beholder.






          share|improve this answer



























            up vote
            2
            down vote













            It's a matriarchy, so women are elites that have all the power. Lots of men all compete to be the best man to produce the next generation of witches, so more men equals more choices.






            share|improve this answer





























              up vote
              2
              down vote













              Lineage



              Men, though unable to use magic, inherit the greater party of parental power.
              Thus boys will be used for arranged marriages against dowry.
              This would allow the greatest witch families to easily perpetuate and increase their power from one generation to the next.
              Introducing the notion of nobility and lineage.
              In the same way arranged marriages will allow to enrich the family, to rise up the ranks of society.



              Moreover, a line that has "lost" its power could rise by marrying a girl to a young boy of a famous lineage



              In conclusion, to answer your question: it would be advantageous to have a single daughter with several brothers, since a boy could be sold married to other families. Boys married or not will not be a burden because they would find work (crafts, soldiers, finances, etc.)






              share|improve this answer





























                up vote
                2
                down vote













                A witch’s time is incredibly valuable. Every minute she’s not casting spells or performing rituals or experimenting with new magic is wasted.



                So men do everything else. Not just the traditional “man’s work” chores like putting up the shelves, but also the traditional “women’s work” chores like cooking and cleaning, and the specialized work like raising the newts and harvesting their eyes, and dealing with the world outside the coven, and even jobs that we’d consider prestigious like engineering that in their world are still not worthy of a witch’s time.



                It’s not about keeping the men down; in fact, “everyone knows” that men are better at cooking and cleaning, as well as math and singing, That isn’t actually true (any more than it’s true that women are better at sweeping and men are better at doctoring in our world), but everyone believes it any way (just as people did in our world until the last century or so).



                The one thing the men can’t do is teach new witches, which is a very time-intensive task. Schooling can only do so much; magic is highly personal, and much of it has to be passed on directly from mother to daughter. If you have four daughters, that quadruples the time you have to spend teaching them, which is time as you have to take away from actual witching. (What happens if a mother dies? Her nearest female relative with no daughters is expected to adopt and raise the girl instead of having one of her own.)



                But if you have one daughter and three sons, that’s three extra people that can be useful to the coven without needing to be trained by a witch, so you’re wasting a lot less time.



                Why couldn’t there just be women who aren’t witches, so they can divide the labor between men and mundane women? Well, you could have a daughter and not raise her to be a witch, but it would be considered horribly uncouth. People would think you’re being unconscionably cruel to your daughter, not to mention depriving the community of a witch.



                Maybe there are some cultures that did develop that way, but every major civilization today traces its descent to a handful of cultures (like the Greek, Hebrew, Indus Valley, and Yellow River cultures in our world) that don’t, so as far as everyone is concerned, this is the way things have always been. The different major civilizations can be different in major ways (e.g., westerners are polyandrous, centralians encourage male homosexuality, easters expect women to be serially monogamous and men compete to be a woman’s last baby-daddy, the one who gets to stay with her through menopause and into eventual retirement), but they all have no place for non-witch women in their sexual politics. The ones who did are thought of like the Philistines or Barbarians of our world.






                share|improve this answer




















                • This sounds like a self weakening argument to me. We don't make witches because they are too valuable
                  – Andrey
                  Aug 30 at 18:52










                • @Andrey No, we don’t make more witches because they take too much training. That’s what the whole fourth paragraph is about.
                  – abarnert
                  Aug 30 at 19:29










                • If it takes more than 1 witch working full time to train a witch you would have 0 witches. If taring witches is a positive outcome you would train as many as you can because you know they are extremely valuable.
                  – Andrey
                  Aug 30 at 20:02

















                up vote
                1
                down vote













                While in theory only one man is needed to get offsprings for good large willage of females, probably (if virginity is not required) powerful one (females) would prefere to have at least choise from man (not only one god or bad) eventually have even each ones harem.






                share|improve this answer



























                  up vote
                  1
                  down vote













                  The amount of mana available to use for magic is divided among the current witches. Too many accessing it's powers will diminish the powers of them all. Having more males ensures that the witches in power stay powerful. Have another female born when a witch is retiring (dying) or has died already in order to bring it back to balance.






                  share|improve this answer





























                    up vote
                    1
                    down vote













                    The society allows for witches to have multiple husbands, so you have lots of male children to marry them off to other covens in order to cement alliances.



                    Marrying off daughters would weaken you, so use sons.



                    Also, everybody has magic - they don't need any particular material good, so you can't "buy" allies the same way you otherwise could. But what's the one thing a sensible witch can't make herself? Men that are safe to have children with. So you trade those.



                    As a final bonus, men can be pretty useful if you don't feel like casting spells to fetch water, build walls, and do other simple tasks.






                    share|improve this answer



























                      up vote
                      1
                      down vote













                      Human sacrifice is required for more complicated spells. The women certainly would not reduce the magic from the gene pool by sacrificing females/witches. That leaves those mostly worthless men as fodder for their dark spells.






                      share|improve this answer



























                        up vote
                        1
                        down vote













                        The Iroquois Confederacy had a rather unique gender role system. Women had more political power, but men were vital to society as well. Generally, Men were used in matters of foreign affairs - Diplomacy and War. When the confederation formed, Men were sent to the confederation council (the Sachem or Grand Sachem) to represent the local tribe's female exclusive council. The local councils had more power technically, because they had the right to recall their representative (called Horn Knocking due to ceremonial importance of deer horns in the headdress among the male leadership). Both men and women councils had to reach a 75% consensus for a decision to pass.



                        In the home front, the women owned the property and it was passed down matrilinearly and men were adopted into the family of their wives and was considered on the level of his wife's brothers in the household. Men were also exclusively warriors but only women had the ability to declare the war. This lead to a rather unique war philosophy called "Mourning Wars" which sought captives to replace deceased members of the family and tribe (often the war dead from a prievious attack or raid). Thus, the goal of wars for the Confederacy was to capture as many people and bring them back alive to be adopted into the tribe, not territorial gains. Thus slaughter of the enemy was seen as a bad tactic as it decreased the success of the war, and the war could be a failure if the number of war dead on their own side was not offset by the captives taken, so the Iroquois were quick to retreat.



                        Those that did not go to war were declared cowards by the clan mother and it was taboo for women to marry men declared as such. Thus, men went to war to prove they were worth of marriage. Typically, a male War Chief was declared who would have authority on the battle field but not much authority at home.



                        All of this existed due to the creation myth, which held that women were stewards of the land and must be respected in their decisions. Men traditionally hunted game while women tended to the farm and the land the clans controlled, and thus had greater importance. Thus their ability to name leaders of the tribes and clans. While men would be the face of the Confederacy to outsiders, women were often the power behind the throne. This could translate into a matrilinear society where the magic of women was such that it was better for them to remain close to the home and not participate in the day to day management of the government by traveling to meeting bodies or court favor from the courts, but the men worked for them, not the reverse, and if they didn't like the outcome, they could remove the offender from office.



                        (It should be pointed out that Iroquois aren't considered a Matriarchal society... both Genders had political power and were quite important in governing the nation.).






                        share|improve this answer



























                          up vote
                          0
                          down vote













                          Marriage as Business



                          In the first minutes of Netflix's Explained chapter about monogamy, the narrator states that some historians believe that marriage was invented as a way to have a "gain" for your family - a means to join forces with another "clan". They give several examples as to how marriage was used to join powerful families and mantain control over the ages (Cleopatra and the romans).



                          The reason for men to exist could be linked to that. If powerful witches have a high social standing, it would be only natural that a certain witch would have a male son to join two powerful "witch houses".



                          Women are Rare



                          Women already play a central role in society without magic powers. They have children and thus, are the only ones capable of perpetuating society.



                          But what if having a female child was really hard?



                          Since they manage to control mana and be very powerful, there could be some magical BS that made women really hard to come by - to the point that witches would actually rely on magic to try and shift those odds for a better chance for a girl to be born.



                          Some people have already said here in here that maybe a newborn witch draws a portion of the mother's power. Maybe that's true and, therefore, only the most powerful witches could have a chance to give birth to a girl - and that would certainly take a toll on the mother.



                          Maybe it could even be a death sentence to try and give birth to a woman!



                          Seventh Son of a Seventh Son



                          I don't know where this saying originated, but it has something to do with great power.



                          Suppose all witches have to be the seventh children - after an unbroken line of six male sons - of a mother who, in turn, is a seventh children herself.



                          This would fairly explain why are there so much more men than women and also why they would rely on magic to have a lot of male children before a woman is born.



                          An Unorthodox Magic Component



                          Having a male children could be a magic component for some desired spell - like eternal life.



                          Suppose there's a ritual that gives you 10 more years to live. Witches would certainly cast this spell multiple times through the years in order to live for centuries.



                          Of course it would have to be a high level spell and, therefore, require a lot of time and resources - and a male children could be one of these resources. In this question, I tried to make a silly game where people would come up with crazy magic components for spells. For this particular spell, one of the main components could be "being pregnant with a boy" or "giving birth to a boy".






                          share|improve this answer




















                          • I like your Seventh Child option, it has roots in a lot of common lore in fiction as well as folklore (for a modern example: In the Harry Potter books, Ginny Weasley is the only daughter and seventh child of the Weasley Family) Seems like an elegant answer because it necessitates a 6:1 ratio of men to women just to keep up the numbers of witches in the community, and a witch would only likely produce one other witch in her life..
                            – Ruadhan
                            Aug 31 at 8:29

















                          up vote
                          0
                          down vote













                          Dowry, which actually caused a gender imbalance in China.



                          Men are expected to perform all of the non-magic chores. This includes gathering things needed by the household, feeding the family members, cleaning the home, and taking care of children and the elderly. Women can therefore focus their efforts on their magic. Although the practice may seem outdated to us, everyone does benefit by it, and it is the norm of that culture.



                          When a couple marries, the husband is expected to care for the wife's aging parents. This means the husband is no longer available to care for his own parents. At the time of marriage, the wife's parents compensate the husband's parents by paying a dowry. The practice is very common in various real-world cultures, although the roles of gender are usually reversed.



                          Something is causing couples to have fewer children. In China it was the One Child policy, but you can make up any reason you want. Supply and demand causes the cost of dowries to radically increase, which favors couples with sons and penalizes couples with daughters. Couples therefore choose to have more sons than daughters.



                          This actually happened in China, which now has 30 million more males than females.






                          share|improve this answer



























                            up vote
                            0
                            down vote













                            There could be a number of reasons,



                            1. Hierarchical Limits: If only women can practice witchcraft then daughters = power. If the covens live in a society, then they are most likely organized and their power is restricted. The most powerful covens are allowed to have the most daughters, lesser covens fewer. The main question with this though is why would they get pregnant if they were 'full' on the number of daughters they could have. Which leads to the next possible reason.


                            2. Maximizing Potential Members: So let's assume that the coven can only support so many members, either from the above scenario, or for some other reason such as available magic or resources. Logically they would want the best and the brightest. Let's suppose The coven can support 9 members but only has 8. Each baby born is examined, only daughters with the highest potential are carried to term, the rest become boys. This leads to an excess of male children.


                            3. Breeding Stock: In this world only women can use witchcraft, but what if the potential to use witchcraft is mostly inherited from the male side? In this scenario then some males might be extremely valuable, and some well, might be more expendable.


                            4. Cannon Fodder: Every army needs foot soldiers. Though from the description of the world Covens would already be on top. Perhaps as muscle to keep the non coven people in line and handle mundane tasks?






                            share|improve this answer



























                              up vote
                              0
                              down vote













                              The Witch tax.



                              Witches are dangerous. Local governments fear nothing more than rogue covenants. Every girl born must be entered into the system. This is expensive. There is a tax collected from every girl's parents. This tax pays for school, social education and frequent visits from the "caretakers". This tax is not cheap.



                              If the parents can't pay the tax the child is drowned. Any attempt to hide a girl is punished by death.






                              share|improve this answer





























                                up vote
                                0
                                down vote













                                Economics and Resources



                                Economics and the availability of resources has a huge impact on real world geopolitics and socials structures no matter which period you are looking at, you could use it in your world to explain at least in part the wanted behavior.



                                This point match with the "numerous ingredients" in your post. If resources aren't infinite in you world, there is probably a competition for resources between witches.



                                Witches would have a huge requirements of ingredients for several reasons :



                                • Casting spells

                                • Training in spells : how can you get better at magic without actual practice ?

                                • Increasing your own magic power : for example through regular consumption of elixir made from expensive components

                                • Building magical tools if they exist in your world (alchemy, wands, golems, magic jewelry, ...)

                                For all the reason listed above, the confirmed witches would already have a huge everyday cost in resources consumption and the training of new witches would also be extremely expensive. It would then be obvious it is better to focus the resources on a select few rather than share them along a lot of witches who will end up half-trained.



                                Chinese wuxia/xianxia novels use this mechanism a lot to drive competition between cultivators (people who train to become immortals like in Chinese myths)



                                If you combine this struggle for resources with other reasons that make it unacceptable to leave women untrained in magic, you get a mechanism to explain the imbalance in the male/female population.



                                This reason could be social stigma as suggested in another answer.



                                Or a stronger practical reason : every woman is born with magic whether trained or not and untrained, and if left unchecked accumulate mana over time, releasing surges of wild magic randomly (maybe only once a certain age threshold is passed).



                                These women would basically be walking time-bombs and witch training for young/teenage girls could easily be made compulsory (at least in civilized parts of the world, I'm not sure how medieval or modern your world is).



                                Combine the mandatory magic training with the limited resources and it could make giving birth to a female child a matter to plan ahead for most witches, making them use the sex choice spell to have male offspring most of the time as males would provide mundane workforce without the resource strain.






                                share|improve this answer





























                                  up vote
                                  0
                                  down vote













                                  How about aging? If a witch is able to prolong her life for hundreds of years, each female offspring represents potential direct competition. If a witch has 2 daughters and in 20 years they are a match for her power what happens if one of them decides they really don't want to wait for mom to die off before grabbing that inheritance. What if they rebel against mommy and join a competing coven? Males, on the other hand, have no chance of affecting mom's power base so are much safer.



                                  Additionally, males though not able to directly use the magic still share half of their genes from their mother. Powerful witches could create alliances in an attempt to improve magic bloodlines by marrying off their male descendants to other powerful witches.






                                  share|improve this answer



















                                    protected by James♦ Aug 31 at 19:04



                                    Thank you for your interest in this question.
                                    Because it has attracted low-quality or spam answers that had to be removed, posting an answer now requires 10 reputation on this site (the association bonus does not count).



                                    Would you like to answer one of these unanswered questions instead?














                                    23 Answers
                                    23






                                    active

                                    oldest

                                    votes








                                    23 Answers
                                    23






                                    active

                                    oldest

                                    votes









                                    active

                                    oldest

                                    votes






                                    active

                                    oldest

                                    votes








                                    up vote
                                    41
                                    down vote













                                    Males can be used as unskilled labor, and you need more unskilled than skilled workers (at least with medieval technology). Or males are used as cannon fodder in wars, or as internal police force, which increases their mortality.



                                    More female offspring:



                                    • Require more teaching effort from mother. B/c untamed magical powers can be very destructive. Magical powers are inherited, and mother is responsible for destruction caused by her child (if she survives, ofc).

                                    • Will fight more between themselves, especially is magical power is limited or shared within a family.

                                    • Are more likely to rebel against the mother. More children mean weaker bond between mother and each child, and higher chance than any one child starts feeling that mother favors other child more than them.

                                    Finally, witchcraft will likely extend life, but only for the witch herself. So to keep gender the ratio constant, you have to have fewer female newborns.



                                    PS For modern science, it is a lot easier to pick gender of the child on conception than to change gender of a growing fetus. If the witch can do same things with her magic, she might as well choose to clone herself rather than mix genes with some male.






                                    share|improve this answer


















                                    • 9




                                      That also feels like a very good argument to make daughters 'daughters of their mother' and sons 'sons of society'. Basically a society could have a lot of structures to teach boys in bulk for their unskilled labor, while the girls are kept with their mother for proper training
                                      – Patrice
                                      Aug 29 at 22:45






                                    • 1




                                      +1 for your remark about different lifespans, this seems like a very good argument!
                                      – Bananenaffe
                                      Aug 30 at 6:00










                                    • @Patrice In this case woman will not have any incentive to have boys. It will be overly complicated and painful way to make abortion.
                                      – talex
                                      Aug 30 at 7:37










                                    • -1: lots of assumptions and most of them I disagree with. Men already have a higher mortality rate without magic - and make up less than 50% of the population because it quite frankly, doesn't help the species survive. One man can reproduce with many women just as effectively as one. This won't be "magically" offset by unskilled labour (you don't need it, just use magic); policing/fighting (how can a non-magic user combat a magic user?). And the idea that magic users would extend life is an assumption challenged in many stories.
                                      – UKMonkey
                                      Aug 30 at 11:55










                                    • This makes sence except that there would be a lot more girls than boys. I think restricting the birth rate/percentage of girls could be quite a cool idea. For example because the mother is a magic user and the girl would inherit the megic a lot of girls born dead (some magic anomaly) while boys seems to be more resistant for this etc.
                                      – Lakatos Gyula
                                      Aug 30 at 12:15














                                    up vote
                                    41
                                    down vote













                                    Males can be used as unskilled labor, and you need more unskilled than skilled workers (at least with medieval technology). Or males are used as cannon fodder in wars, or as internal police force, which increases their mortality.



                                    More female offspring:



                                    • Require more teaching effort from mother. B/c untamed magical powers can be very destructive. Magical powers are inherited, and mother is responsible for destruction caused by her child (if she survives, ofc).

                                    • Will fight more between themselves, especially is magical power is limited or shared within a family.

                                    • Are more likely to rebel against the mother. More children mean weaker bond between mother and each child, and higher chance than any one child starts feeling that mother favors other child more than them.

                                    Finally, witchcraft will likely extend life, but only for the witch herself. So to keep gender the ratio constant, you have to have fewer female newborns.



                                    PS For modern science, it is a lot easier to pick gender of the child on conception than to change gender of a growing fetus. If the witch can do same things with her magic, she might as well choose to clone herself rather than mix genes with some male.






                                    share|improve this answer


















                                    • 9




                                      That also feels like a very good argument to make daughters 'daughters of their mother' and sons 'sons of society'. Basically a society could have a lot of structures to teach boys in bulk for their unskilled labor, while the girls are kept with their mother for proper training
                                      – Patrice
                                      Aug 29 at 22:45






                                    • 1




                                      +1 for your remark about different lifespans, this seems like a very good argument!
                                      – Bananenaffe
                                      Aug 30 at 6:00










                                    • @Patrice In this case woman will not have any incentive to have boys. It will be overly complicated and painful way to make abortion.
                                      – talex
                                      Aug 30 at 7:37










                                    • -1: lots of assumptions and most of them I disagree with. Men already have a higher mortality rate without magic - and make up less than 50% of the population because it quite frankly, doesn't help the species survive. One man can reproduce with many women just as effectively as one. This won't be "magically" offset by unskilled labour (you don't need it, just use magic); policing/fighting (how can a non-magic user combat a magic user?). And the idea that magic users would extend life is an assumption challenged in many stories.
                                      – UKMonkey
                                      Aug 30 at 11:55










                                    • This makes sence except that there would be a lot more girls than boys. I think restricting the birth rate/percentage of girls could be quite a cool idea. For example because the mother is a magic user and the girl would inherit the megic a lot of girls born dead (some magic anomaly) while boys seems to be more resistant for this etc.
                                      – Lakatos Gyula
                                      Aug 30 at 12:15












                                    up vote
                                    41
                                    down vote










                                    up vote
                                    41
                                    down vote









                                    Males can be used as unskilled labor, and you need more unskilled than skilled workers (at least with medieval technology). Or males are used as cannon fodder in wars, or as internal police force, which increases their mortality.



                                    More female offspring:



                                    • Require more teaching effort from mother. B/c untamed magical powers can be very destructive. Magical powers are inherited, and mother is responsible for destruction caused by her child (if she survives, ofc).

                                    • Will fight more between themselves, especially is magical power is limited or shared within a family.

                                    • Are more likely to rebel against the mother. More children mean weaker bond between mother and each child, and higher chance than any one child starts feeling that mother favors other child more than them.

                                    Finally, witchcraft will likely extend life, but only for the witch herself. So to keep gender the ratio constant, you have to have fewer female newborns.



                                    PS For modern science, it is a lot easier to pick gender of the child on conception than to change gender of a growing fetus. If the witch can do same things with her magic, she might as well choose to clone herself rather than mix genes with some male.






                                    share|improve this answer














                                    Males can be used as unskilled labor, and you need more unskilled than skilled workers (at least with medieval technology). Or males are used as cannon fodder in wars, or as internal police force, which increases their mortality.



                                    More female offspring:



                                    • Require more teaching effort from mother. B/c untamed magical powers can be very destructive. Magical powers are inherited, and mother is responsible for destruction caused by her child (if she survives, ofc).

                                    • Will fight more between themselves, especially is magical power is limited or shared within a family.

                                    • Are more likely to rebel against the mother. More children mean weaker bond between mother and each child, and higher chance than any one child starts feeling that mother favors other child more than them.

                                    Finally, witchcraft will likely extend life, but only for the witch herself. So to keep gender the ratio constant, you have to have fewer female newborns.



                                    PS For modern science, it is a lot easier to pick gender of the child on conception than to change gender of a growing fetus. If the witch can do same things with her magic, she might as well choose to clone herself rather than mix genes with some male.







                                    share|improve this answer














                                    share|improve this answer



                                    share|improve this answer








                                    edited Aug 29 at 19:07

























                                    answered Aug 29 at 18:54









                                    Bald Bear

                                    3,420513




                                    3,420513







                                    • 9




                                      That also feels like a very good argument to make daughters 'daughters of their mother' and sons 'sons of society'. Basically a society could have a lot of structures to teach boys in bulk for their unskilled labor, while the girls are kept with their mother for proper training
                                      – Patrice
                                      Aug 29 at 22:45






                                    • 1




                                      +1 for your remark about different lifespans, this seems like a very good argument!
                                      – Bananenaffe
                                      Aug 30 at 6:00










                                    • @Patrice In this case woman will not have any incentive to have boys. It will be overly complicated and painful way to make abortion.
                                      – talex
                                      Aug 30 at 7:37










                                    • -1: lots of assumptions and most of them I disagree with. Men already have a higher mortality rate without magic - and make up less than 50% of the population because it quite frankly, doesn't help the species survive. One man can reproduce with many women just as effectively as one. This won't be "magically" offset by unskilled labour (you don't need it, just use magic); policing/fighting (how can a non-magic user combat a magic user?). And the idea that magic users would extend life is an assumption challenged in many stories.
                                      – UKMonkey
                                      Aug 30 at 11:55










                                    • This makes sence except that there would be a lot more girls than boys. I think restricting the birth rate/percentage of girls could be quite a cool idea. For example because the mother is a magic user and the girl would inherit the megic a lot of girls born dead (some magic anomaly) while boys seems to be more resistant for this etc.
                                      – Lakatos Gyula
                                      Aug 30 at 12:15












                                    • 9




                                      That also feels like a very good argument to make daughters 'daughters of their mother' and sons 'sons of society'. Basically a society could have a lot of structures to teach boys in bulk for their unskilled labor, while the girls are kept with their mother for proper training
                                      – Patrice
                                      Aug 29 at 22:45






                                    • 1




                                      +1 for your remark about different lifespans, this seems like a very good argument!
                                      – Bananenaffe
                                      Aug 30 at 6:00










                                    • @Patrice In this case woman will not have any incentive to have boys. It will be overly complicated and painful way to make abortion.
                                      – talex
                                      Aug 30 at 7:37










                                    • -1: lots of assumptions and most of them I disagree with. Men already have a higher mortality rate without magic - and make up less than 50% of the population because it quite frankly, doesn't help the species survive. One man can reproduce with many women just as effectively as one. This won't be "magically" offset by unskilled labour (you don't need it, just use magic); policing/fighting (how can a non-magic user combat a magic user?). And the idea that magic users would extend life is an assumption challenged in many stories.
                                      – UKMonkey
                                      Aug 30 at 11:55










                                    • This makes sence except that there would be a lot more girls than boys. I think restricting the birth rate/percentage of girls could be quite a cool idea. For example because the mother is a magic user and the girl would inherit the megic a lot of girls born dead (some magic anomaly) while boys seems to be more resistant for this etc.
                                      – Lakatos Gyula
                                      Aug 30 at 12:15







                                    9




                                    9




                                    That also feels like a very good argument to make daughters 'daughters of their mother' and sons 'sons of society'. Basically a society could have a lot of structures to teach boys in bulk for their unskilled labor, while the girls are kept with their mother for proper training
                                    – Patrice
                                    Aug 29 at 22:45




                                    That also feels like a very good argument to make daughters 'daughters of their mother' and sons 'sons of society'. Basically a society could have a lot of structures to teach boys in bulk for their unskilled labor, while the girls are kept with their mother for proper training
                                    – Patrice
                                    Aug 29 at 22:45




                                    1




                                    1




                                    +1 for your remark about different lifespans, this seems like a very good argument!
                                    – Bananenaffe
                                    Aug 30 at 6:00




                                    +1 for your remark about different lifespans, this seems like a very good argument!
                                    – Bananenaffe
                                    Aug 30 at 6:00












                                    @Patrice In this case woman will not have any incentive to have boys. It will be overly complicated and painful way to make abortion.
                                    – talex
                                    Aug 30 at 7:37




                                    @Patrice In this case woman will not have any incentive to have boys. It will be overly complicated and painful way to make abortion.
                                    – talex
                                    Aug 30 at 7:37












                                    -1: lots of assumptions and most of them I disagree with. Men already have a higher mortality rate without magic - and make up less than 50% of the population because it quite frankly, doesn't help the species survive. One man can reproduce with many women just as effectively as one. This won't be "magically" offset by unskilled labour (you don't need it, just use magic); policing/fighting (how can a non-magic user combat a magic user?). And the idea that magic users would extend life is an assumption challenged in many stories.
                                    – UKMonkey
                                    Aug 30 at 11:55




                                    -1: lots of assumptions and most of them I disagree with. Men already have a higher mortality rate without magic - and make up less than 50% of the population because it quite frankly, doesn't help the species survive. One man can reproduce with many women just as effectively as one. This won't be "magically" offset by unskilled labour (you don't need it, just use magic); policing/fighting (how can a non-magic user combat a magic user?). And the idea that magic users would extend life is an assumption challenged in many stories.
                                    – UKMonkey
                                    Aug 30 at 11:55












                                    This makes sence except that there would be a lot more girls than boys. I think restricting the birth rate/percentage of girls could be quite a cool idea. For example because the mother is a magic user and the girl would inherit the megic a lot of girls born dead (some magic anomaly) while boys seems to be more resistant for this etc.
                                    – Lakatos Gyula
                                    Aug 30 at 12:15




                                    This makes sence except that there would be a lot more girls than boys. I think restricting the birth rate/percentage of girls could be quite a cool idea. For example because the mother is a magic user and the girl would inherit the megic a lot of girls born dead (some magic anomaly) while boys seems to be more resistant for this etc.
                                    – Lakatos Gyula
                                    Aug 30 at 12:15










                                    up vote
                                    18
                                    down vote













                                    Witch offspring sap mana from their mothers.



                                    You can have a witch baby but when she emerges she takes some of mother's magic with her. If you want to be at the top of your magic game you do not want to share your magic with your offspring. The uberwitches might have only boys - but then their lineage dies. Middle powered witches might opt to share with their daughters because the lineage benefit outweighs the power loss.






                                    share|improve this answer


















                                    • 3




                                      I like this answer, particularly since it dovetails neatly with the Shakespearian/Discworld paradigm of "Maiden, Mother and Crone"
                                      – Ruadhan
                                      Aug 30 at 12:35










                                    • @Ruadhan What does that mean?
                                      – Incognito
                                      Aug 30 at 16:51






                                    • 2




                                      Traditionally there are three witches (eg: Macbeth),which in the Discworld books represent three major age-groups, the young maiden, the middle-aged mother and the old-aged "crone" So in WillK's scenario, the community would naturally split into that pattern because choosing to be a parent would have a direct impact on magical power. You'd naturally in a given group have more powerful adult witches with no heirs, middle-powered Mother witches and younger Maidens growing into their power, likely the child of the Mother. Honestly I think I'm talking out of my arse at this point though :P
                                      – Ruadhan
                                      Aug 31 at 8:22














                                    up vote
                                    18
                                    down vote













                                    Witch offspring sap mana from their mothers.



                                    You can have a witch baby but when she emerges she takes some of mother's magic with her. If you want to be at the top of your magic game you do not want to share your magic with your offspring. The uberwitches might have only boys - but then their lineage dies. Middle powered witches might opt to share with their daughters because the lineage benefit outweighs the power loss.






                                    share|improve this answer


















                                    • 3




                                      I like this answer, particularly since it dovetails neatly with the Shakespearian/Discworld paradigm of "Maiden, Mother and Crone"
                                      – Ruadhan
                                      Aug 30 at 12:35










                                    • @Ruadhan What does that mean?
                                      – Incognito
                                      Aug 30 at 16:51






                                    • 2




                                      Traditionally there are three witches (eg: Macbeth),which in the Discworld books represent three major age-groups, the young maiden, the middle-aged mother and the old-aged "crone" So in WillK's scenario, the community would naturally split into that pattern because choosing to be a parent would have a direct impact on magical power. You'd naturally in a given group have more powerful adult witches with no heirs, middle-powered Mother witches and younger Maidens growing into their power, likely the child of the Mother. Honestly I think I'm talking out of my arse at this point though :P
                                      – Ruadhan
                                      Aug 31 at 8:22












                                    up vote
                                    18
                                    down vote










                                    up vote
                                    18
                                    down vote









                                    Witch offspring sap mana from their mothers.



                                    You can have a witch baby but when she emerges she takes some of mother's magic with her. If you want to be at the top of your magic game you do not want to share your magic with your offspring. The uberwitches might have only boys - but then their lineage dies. Middle powered witches might opt to share with their daughters because the lineage benefit outweighs the power loss.






                                    share|improve this answer














                                    Witch offspring sap mana from their mothers.



                                    You can have a witch baby but when she emerges she takes some of mother's magic with her. If you want to be at the top of your magic game you do not want to share your magic with your offspring. The uberwitches might have only boys - but then their lineage dies. Middle powered witches might opt to share with their daughters because the lineage benefit outweighs the power loss.







                                    share|improve this answer














                                    share|improve this answer



                                    share|improve this answer








                                    edited Aug 29 at 21:55

























                                    answered Aug 29 at 21:17









                                    Willk

                                    85.9k21170373




                                    85.9k21170373







                                    • 3




                                      I like this answer, particularly since it dovetails neatly with the Shakespearian/Discworld paradigm of "Maiden, Mother and Crone"
                                      – Ruadhan
                                      Aug 30 at 12:35










                                    • @Ruadhan What does that mean?
                                      – Incognito
                                      Aug 30 at 16:51






                                    • 2




                                      Traditionally there are three witches (eg: Macbeth),which in the Discworld books represent three major age-groups, the young maiden, the middle-aged mother and the old-aged "crone" So in WillK's scenario, the community would naturally split into that pattern because choosing to be a parent would have a direct impact on magical power. You'd naturally in a given group have more powerful adult witches with no heirs, middle-powered Mother witches and younger Maidens growing into their power, likely the child of the Mother. Honestly I think I'm talking out of my arse at this point though :P
                                      – Ruadhan
                                      Aug 31 at 8:22












                                    • 3




                                      I like this answer, particularly since it dovetails neatly with the Shakespearian/Discworld paradigm of "Maiden, Mother and Crone"
                                      – Ruadhan
                                      Aug 30 at 12:35










                                    • @Ruadhan What does that mean?
                                      – Incognito
                                      Aug 30 at 16:51






                                    • 2




                                      Traditionally there are three witches (eg: Macbeth),which in the Discworld books represent three major age-groups, the young maiden, the middle-aged mother and the old-aged "crone" So in WillK's scenario, the community would naturally split into that pattern because choosing to be a parent would have a direct impact on magical power. You'd naturally in a given group have more powerful adult witches with no heirs, middle-powered Mother witches and younger Maidens growing into their power, likely the child of the Mother. Honestly I think I'm talking out of my arse at this point though :P
                                      – Ruadhan
                                      Aug 31 at 8:22







                                    3




                                    3




                                    I like this answer, particularly since it dovetails neatly with the Shakespearian/Discworld paradigm of "Maiden, Mother and Crone"
                                    – Ruadhan
                                    Aug 30 at 12:35




                                    I like this answer, particularly since it dovetails neatly with the Shakespearian/Discworld paradigm of "Maiden, Mother and Crone"
                                    – Ruadhan
                                    Aug 30 at 12:35












                                    @Ruadhan What does that mean?
                                    – Incognito
                                    Aug 30 at 16:51




                                    @Ruadhan What does that mean?
                                    – Incognito
                                    Aug 30 at 16:51




                                    2




                                    2




                                    Traditionally there are three witches (eg: Macbeth),which in the Discworld books represent three major age-groups, the young maiden, the middle-aged mother and the old-aged "crone" So in WillK's scenario, the community would naturally split into that pattern because choosing to be a parent would have a direct impact on magical power. You'd naturally in a given group have more powerful adult witches with no heirs, middle-powered Mother witches and younger Maidens growing into their power, likely the child of the Mother. Honestly I think I'm talking out of my arse at this point though :P
                                    – Ruadhan
                                    Aug 31 at 8:22




                                    Traditionally there are three witches (eg: Macbeth),which in the Discworld books represent three major age-groups, the young maiden, the middle-aged mother and the old-aged "crone" So in WillK's scenario, the community would naturally split into that pattern because choosing to be a parent would have a direct impact on magical power. You'd naturally in a given group have more powerful adult witches with no heirs, middle-powered Mother witches and younger Maidens growing into their power, likely the child of the Mother. Honestly I think I'm talking out of my arse at this point though :P
                                    – Ruadhan
                                    Aug 31 at 8:22










                                    up vote
                                    11
                                    down vote













                                    Inheritance is matrilineal. Having multiple daughters means that there is some controversy over who inherits. And even without controversy, there is complexity. Primogeniture might give the bulk to the firstborn, but how to provide for second-born and later daughters?



                                    Sons are outside inheritance. Have as many as you want. Their purpose is to make good marriages or to support their sister. Perhaps the occasional dark magic ritual sacrifice to increase power. Sons increase the family power with less concern about splitting it.



                                    One daughter makes it clear who the heir is. A second muddies those waters. And can become an aunt who might claim precedence over a niece.






                                    share|improve this answer
























                                      up vote
                                      11
                                      down vote













                                      Inheritance is matrilineal. Having multiple daughters means that there is some controversy over who inherits. And even without controversy, there is complexity. Primogeniture might give the bulk to the firstborn, but how to provide for second-born and later daughters?



                                      Sons are outside inheritance. Have as many as you want. Their purpose is to make good marriages or to support their sister. Perhaps the occasional dark magic ritual sacrifice to increase power. Sons increase the family power with less concern about splitting it.



                                      One daughter makes it clear who the heir is. A second muddies those waters. And can become an aunt who might claim precedence over a niece.






                                      share|improve this answer






















                                        up vote
                                        11
                                        down vote










                                        up vote
                                        11
                                        down vote









                                        Inheritance is matrilineal. Having multiple daughters means that there is some controversy over who inherits. And even without controversy, there is complexity. Primogeniture might give the bulk to the firstborn, but how to provide for second-born and later daughters?



                                        Sons are outside inheritance. Have as many as you want. Their purpose is to make good marriages or to support their sister. Perhaps the occasional dark magic ritual sacrifice to increase power. Sons increase the family power with less concern about splitting it.



                                        One daughter makes it clear who the heir is. A second muddies those waters. And can become an aunt who might claim precedence over a niece.






                                        share|improve this answer












                                        Inheritance is matrilineal. Having multiple daughters means that there is some controversy over who inherits. And even without controversy, there is complexity. Primogeniture might give the bulk to the firstborn, but how to provide for second-born and later daughters?



                                        Sons are outside inheritance. Have as many as you want. Their purpose is to make good marriages or to support their sister. Perhaps the occasional dark magic ritual sacrifice to increase power. Sons increase the family power with less concern about splitting it.



                                        One daughter makes it clear who the heir is. A second muddies those waters. And can become an aunt who might claim precedence over a niece.







                                        share|improve this answer












                                        share|improve this answer



                                        share|improve this answer










                                        answered Aug 30 at 1:03









                                        Brythan

                                        19.5k74182




                                        19.5k74182




















                                            up vote
                                            7
                                            down vote













                                            The bodies of men collect and concentrate the manna of the world (making more potent spells possible), but they have no ability to control or direct it. A woman will therefore become more powerful herself if she has a few sons before having daughters as she will be able to use their concentrated manna to power her spells.



                                            Please note that a civilization can't make choices, only the individuals in the civilization can, so the choice has to make sense in a per person context.






                                            share|improve this answer
























                                              up vote
                                              7
                                              down vote













                                              The bodies of men collect and concentrate the manna of the world (making more potent spells possible), but they have no ability to control or direct it. A woman will therefore become more powerful herself if she has a few sons before having daughters as she will be able to use their concentrated manna to power her spells.



                                              Please note that a civilization can't make choices, only the individuals in the civilization can, so the choice has to make sense in a per person context.






                                              share|improve this answer






















                                                up vote
                                                7
                                                down vote










                                                up vote
                                                7
                                                down vote









                                                The bodies of men collect and concentrate the manna of the world (making more potent spells possible), but they have no ability to control or direct it. A woman will therefore become more powerful herself if she has a few sons before having daughters as she will be able to use their concentrated manna to power her spells.



                                                Please note that a civilization can't make choices, only the individuals in the civilization can, so the choice has to make sense in a per person context.






                                                share|improve this answer












                                                The bodies of men collect and concentrate the manna of the world (making more potent spells possible), but they have no ability to control or direct it. A woman will therefore become more powerful herself if she has a few sons before having daughters as she will be able to use their concentrated manna to power her spells.



                                                Please note that a civilization can't make choices, only the individuals in the civilization can, so the choice has to make sense in a per person context.







                                                share|improve this answer












                                                share|improve this answer



                                                share|improve this answer










                                                answered Aug 29 at 21:29









                                                Mathaddict

                                                71711




                                                71711




















                                                    up vote
                                                    5
                                                    down vote













                                                    Magic is sex-based, so the power is based on the amount of sex. A woman can have more sex than a man can, so having more men increases the total power.






                                                    share|improve this answer
















                                                    • 1




                                                      Why does the sex have to be with a witch-born male?
                                                      – RonJohn
                                                      Aug 29 at 21:24










                                                    • On what basis can she do t hat?
                                                      – Azor Ahai
                                                      Aug 29 at 23:18






                                                    • 1




                                                      @AzorAhai Women do not have a refractory period. Men can only have sex if they can maintain an erection, which they cannot do 100% of the time, while women can (strictly mechanically speaking) have sex whenever--24 hours a day, if they want.
                                                      – Logan R. Kearsley
                                                      Aug 31 at 12:19














                                                    up vote
                                                    5
                                                    down vote













                                                    Magic is sex-based, so the power is based on the amount of sex. A woman can have more sex than a man can, so having more men increases the total power.






                                                    share|improve this answer
















                                                    • 1




                                                      Why does the sex have to be with a witch-born male?
                                                      – RonJohn
                                                      Aug 29 at 21:24










                                                    • On what basis can she do t hat?
                                                      – Azor Ahai
                                                      Aug 29 at 23:18






                                                    • 1




                                                      @AzorAhai Women do not have a refractory period. Men can only have sex if they can maintain an erection, which they cannot do 100% of the time, while women can (strictly mechanically speaking) have sex whenever--24 hours a day, if they want.
                                                      – Logan R. Kearsley
                                                      Aug 31 at 12:19












                                                    up vote
                                                    5
                                                    down vote










                                                    up vote
                                                    5
                                                    down vote









                                                    Magic is sex-based, so the power is based on the amount of sex. A woman can have more sex than a man can, so having more men increases the total power.






                                                    share|improve this answer












                                                    Magic is sex-based, so the power is based on the amount of sex. A woman can have more sex than a man can, so having more men increases the total power.







                                                    share|improve this answer












                                                    share|improve this answer



                                                    share|improve this answer










                                                    answered Aug 29 at 21:18









                                                    David Thornley

                                                    4514




                                                    4514







                                                    • 1




                                                      Why does the sex have to be with a witch-born male?
                                                      – RonJohn
                                                      Aug 29 at 21:24










                                                    • On what basis can she do t hat?
                                                      – Azor Ahai
                                                      Aug 29 at 23:18






                                                    • 1




                                                      @AzorAhai Women do not have a refractory period. Men can only have sex if they can maintain an erection, which they cannot do 100% of the time, while women can (strictly mechanically speaking) have sex whenever--24 hours a day, if they want.
                                                      – Logan R. Kearsley
                                                      Aug 31 at 12:19












                                                    • 1




                                                      Why does the sex have to be with a witch-born male?
                                                      – RonJohn
                                                      Aug 29 at 21:24










                                                    • On what basis can she do t hat?
                                                      – Azor Ahai
                                                      Aug 29 at 23:18






                                                    • 1




                                                      @AzorAhai Women do not have a refractory period. Men can only have sex if they can maintain an erection, which they cannot do 100% of the time, while women can (strictly mechanically speaking) have sex whenever--24 hours a day, if they want.
                                                      – Logan R. Kearsley
                                                      Aug 31 at 12:19







                                                    1




                                                    1




                                                    Why does the sex have to be with a witch-born male?
                                                    – RonJohn
                                                    Aug 29 at 21:24




                                                    Why does the sex have to be with a witch-born male?
                                                    – RonJohn
                                                    Aug 29 at 21:24












                                                    On what basis can she do t hat?
                                                    – Azor Ahai
                                                    Aug 29 at 23:18




                                                    On what basis can she do t hat?
                                                    – Azor Ahai
                                                    Aug 29 at 23:18




                                                    1




                                                    1




                                                    @AzorAhai Women do not have a refractory period. Men can only have sex if they can maintain an erection, which they cannot do 100% of the time, while women can (strictly mechanically speaking) have sex whenever--24 hours a day, if they want.
                                                    – Logan R. Kearsley
                                                    Aug 31 at 12:19




                                                    @AzorAhai Women do not have a refractory period. Men can only have sex if they can maintain an erection, which they cannot do 100% of the time, while women can (strictly mechanically speaking) have sex whenever--24 hours a day, if they want.
                                                    – Logan R. Kearsley
                                                    Aug 31 at 12:19










                                                    up vote
                                                    4
                                                    down vote













                                                    Males born of witches can't access magic, but they are immune to magic by virtue of their lineage and a quirk of how magic works.



                                                    Thus, they're good -- and devoted -- warriors in the internecine battles between covens.






                                                    share|improve this answer




















                                                    • Males might be immune to magic spells themselves, but they're most likely not immune to damage from objects being moved by magic. Imagine what a bunch of rocks propelled through the air by magic, fast as the hawk flies, can do to a group of men charging at you. And since they're immune to magic, you can't cast defensive spells on them. Or just use a ritual to swallow them up into the earth.
                                                      – Nzall
                                                      Aug 30 at 9:37











                                                    • @Nzall interesting points. Their witch mothers & sisters could create magic shields in front and under them.
                                                      – RonJohn
                                                      Aug 30 at 12:29










                                                    • I was just trying to clarify how being immune to magic does not mean you can't be harmed by a magic user. There are loads of ways for magic to affect a world in such a way that it becomes lethal even to people immune to magic. In the Avatar Cartoon, an Airbender once sucked the air out of a room, killing everyone in it. In Codex Alera, a group of Earthcrafters pulled all the oil and coal to right below the surface which was then ignited by Firecrafters. Shortly after, a watercrafter diverted a river to that field. Magic immunity can be both useful and dangerous.
                                                      – Nzall
                                                      Aug 30 at 13:03










                                                    • @Nzall anything you can do to a magic immune, you can also do to the magic susceptible; the reverse is not true. Thus, being immune to magic is better -- in this case -- than being susceptible. It doesn't make you immortal, though, and should not make you arrogant.
                                                      – RonJohn
                                                      Aug 30 at 14:29














                                                    up vote
                                                    4
                                                    down vote













                                                    Males born of witches can't access magic, but they are immune to magic by virtue of their lineage and a quirk of how magic works.



                                                    Thus, they're good -- and devoted -- warriors in the internecine battles between covens.






                                                    share|improve this answer




















                                                    • Males might be immune to magic spells themselves, but they're most likely not immune to damage from objects being moved by magic. Imagine what a bunch of rocks propelled through the air by magic, fast as the hawk flies, can do to a group of men charging at you. And since they're immune to magic, you can't cast defensive spells on them. Or just use a ritual to swallow them up into the earth.
                                                      – Nzall
                                                      Aug 30 at 9:37











                                                    • @Nzall interesting points. Their witch mothers & sisters could create magic shields in front and under them.
                                                      – RonJohn
                                                      Aug 30 at 12:29










                                                    • I was just trying to clarify how being immune to magic does not mean you can't be harmed by a magic user. There are loads of ways for magic to affect a world in such a way that it becomes lethal even to people immune to magic. In the Avatar Cartoon, an Airbender once sucked the air out of a room, killing everyone in it. In Codex Alera, a group of Earthcrafters pulled all the oil and coal to right below the surface which was then ignited by Firecrafters. Shortly after, a watercrafter diverted a river to that field. Magic immunity can be both useful and dangerous.
                                                      – Nzall
                                                      Aug 30 at 13:03










                                                    • @Nzall anything you can do to a magic immune, you can also do to the magic susceptible; the reverse is not true. Thus, being immune to magic is better -- in this case -- than being susceptible. It doesn't make you immortal, though, and should not make you arrogant.
                                                      – RonJohn
                                                      Aug 30 at 14:29












                                                    up vote
                                                    4
                                                    down vote










                                                    up vote
                                                    4
                                                    down vote









                                                    Males born of witches can't access magic, but they are immune to magic by virtue of their lineage and a quirk of how magic works.



                                                    Thus, they're good -- and devoted -- warriors in the internecine battles between covens.






                                                    share|improve this answer












                                                    Males born of witches can't access magic, but they are immune to magic by virtue of their lineage and a quirk of how magic works.



                                                    Thus, they're good -- and devoted -- warriors in the internecine battles between covens.







                                                    share|improve this answer












                                                    share|improve this answer



                                                    share|improve this answer










                                                    answered Aug 29 at 18:58









                                                    RonJohn

                                                    11.5k12657




                                                    11.5k12657











                                                    • Males might be immune to magic spells themselves, but they're most likely not immune to damage from objects being moved by magic. Imagine what a bunch of rocks propelled through the air by magic, fast as the hawk flies, can do to a group of men charging at you. And since they're immune to magic, you can't cast defensive spells on them. Or just use a ritual to swallow them up into the earth.
                                                      – Nzall
                                                      Aug 30 at 9:37











                                                    • @Nzall interesting points. Their witch mothers & sisters could create magic shields in front and under them.
                                                      – RonJohn
                                                      Aug 30 at 12:29










                                                    • I was just trying to clarify how being immune to magic does not mean you can't be harmed by a magic user. There are loads of ways for magic to affect a world in such a way that it becomes lethal even to people immune to magic. In the Avatar Cartoon, an Airbender once sucked the air out of a room, killing everyone in it. In Codex Alera, a group of Earthcrafters pulled all the oil and coal to right below the surface which was then ignited by Firecrafters. Shortly after, a watercrafter diverted a river to that field. Magic immunity can be both useful and dangerous.
                                                      – Nzall
                                                      Aug 30 at 13:03










                                                    • @Nzall anything you can do to a magic immune, you can also do to the magic susceptible; the reverse is not true. Thus, being immune to magic is better -- in this case -- than being susceptible. It doesn't make you immortal, though, and should not make you arrogant.
                                                      – RonJohn
                                                      Aug 30 at 14:29
















                                                    • Males might be immune to magic spells themselves, but they're most likely not immune to damage from objects being moved by magic. Imagine what a bunch of rocks propelled through the air by magic, fast as the hawk flies, can do to a group of men charging at you. And since they're immune to magic, you can't cast defensive spells on them. Or just use a ritual to swallow them up into the earth.
                                                      – Nzall
                                                      Aug 30 at 9:37











                                                    • @Nzall interesting points. Their witch mothers & sisters could create magic shields in front and under them.
                                                      – RonJohn
                                                      Aug 30 at 12:29










                                                    • I was just trying to clarify how being immune to magic does not mean you can't be harmed by a magic user. There are loads of ways for magic to affect a world in such a way that it becomes lethal even to people immune to magic. In the Avatar Cartoon, an Airbender once sucked the air out of a room, killing everyone in it. In Codex Alera, a group of Earthcrafters pulled all the oil and coal to right below the surface which was then ignited by Firecrafters. Shortly after, a watercrafter diverted a river to that field. Magic immunity can be both useful and dangerous.
                                                      – Nzall
                                                      Aug 30 at 13:03










                                                    • @Nzall anything you can do to a magic immune, you can also do to the magic susceptible; the reverse is not true. Thus, being immune to magic is better -- in this case -- than being susceptible. It doesn't make you immortal, though, and should not make you arrogant.
                                                      – RonJohn
                                                      Aug 30 at 14:29















                                                    Males might be immune to magic spells themselves, but they're most likely not immune to damage from objects being moved by magic. Imagine what a bunch of rocks propelled through the air by magic, fast as the hawk flies, can do to a group of men charging at you. And since they're immune to magic, you can't cast defensive spells on them. Or just use a ritual to swallow them up into the earth.
                                                    – Nzall
                                                    Aug 30 at 9:37





                                                    Males might be immune to magic spells themselves, but they're most likely not immune to damage from objects being moved by magic. Imagine what a bunch of rocks propelled through the air by magic, fast as the hawk flies, can do to a group of men charging at you. And since they're immune to magic, you can't cast defensive spells on them. Or just use a ritual to swallow them up into the earth.
                                                    – Nzall
                                                    Aug 30 at 9:37













                                                    @Nzall interesting points. Their witch mothers & sisters could create magic shields in front and under them.
                                                    – RonJohn
                                                    Aug 30 at 12:29




                                                    @Nzall interesting points. Their witch mothers & sisters could create magic shields in front and under them.
                                                    – RonJohn
                                                    Aug 30 at 12:29












                                                    I was just trying to clarify how being immune to magic does not mean you can't be harmed by a magic user. There are loads of ways for magic to affect a world in such a way that it becomes lethal even to people immune to magic. In the Avatar Cartoon, an Airbender once sucked the air out of a room, killing everyone in it. In Codex Alera, a group of Earthcrafters pulled all the oil and coal to right below the surface which was then ignited by Firecrafters. Shortly after, a watercrafter diverted a river to that field. Magic immunity can be both useful and dangerous.
                                                    – Nzall
                                                    Aug 30 at 13:03




                                                    I was just trying to clarify how being immune to magic does not mean you can't be harmed by a magic user. There are loads of ways for magic to affect a world in such a way that it becomes lethal even to people immune to magic. In the Avatar Cartoon, an Airbender once sucked the air out of a room, killing everyone in it. In Codex Alera, a group of Earthcrafters pulled all the oil and coal to right below the surface which was then ignited by Firecrafters. Shortly after, a watercrafter diverted a river to that field. Magic immunity can be both useful and dangerous.
                                                    – Nzall
                                                    Aug 30 at 13:03












                                                    @Nzall anything you can do to a magic immune, you can also do to the magic susceptible; the reverse is not true. Thus, being immune to magic is better -- in this case -- than being susceptible. It doesn't make you immortal, though, and should not make you arrogant.
                                                    – RonJohn
                                                    Aug 30 at 14:29




                                                    @Nzall anything you can do to a magic immune, you can also do to the magic susceptible; the reverse is not true. Thus, being immune to magic is better -- in this case -- than being susceptible. It doesn't make you immortal, though, and should not make you arrogant.
                                                    – RonJohn
                                                    Aug 30 at 14:29










                                                    up vote
                                                    4
                                                    down vote













                                                    Untrained witches are dangerous to everyone. And you can train only so many at once.



                                                    Every daughter of a witch has magic. But it is training that makes a witch out of a girl. Without training, magic is still there, but it is spontaneous, wild, bursting out at random. None wants to be around an untrained witch, as you have to maintain strongest protective charms at all time and still won't be completely sure if that would be enough.



                                                    On the other hand, the training involved is long and deeply personal. One full-fledged witch can train only one apprentice at a time. And the training up to the moment when she can be graduated and the next apprentice gets accepted takes two decades. Thus, it is extremely rare for a witch to mentor more than two girls over her lifetime (she would graduate the second one in her sixties at the earliest). And many elect to train only a single designated successor.



                                                    Of course, basic control gets taught early on, and you can have a non-volatile girl just a year after her magic manifests. Yet, without further training she wouldn't contribute much to the coven, and it would be such a waste (especially for her mentor) to just cut training short there. And if an unclaimed girl comes of age for whatever reason (say, her designated mentor met an accident), then either someone free (and unwilling) has to teach her, or someone has to halt teaching their current apprentice to deal with a new menace.



                                                    Thus, it's not that boys are preferred. But there is a hard limit on how many girls are acceptable, and everyone else has to be boys. Assuming witches aren't too chaste, yet don't use contraception, this will be a natural consequence.






                                                    share|improve this answer
























                                                      up vote
                                                      4
                                                      down vote













                                                      Untrained witches are dangerous to everyone. And you can train only so many at once.



                                                      Every daughter of a witch has magic. But it is training that makes a witch out of a girl. Without training, magic is still there, but it is spontaneous, wild, bursting out at random. None wants to be around an untrained witch, as you have to maintain strongest protective charms at all time and still won't be completely sure if that would be enough.



                                                      On the other hand, the training involved is long and deeply personal. One full-fledged witch can train only one apprentice at a time. And the training up to the moment when she can be graduated and the next apprentice gets accepted takes two decades. Thus, it is extremely rare for a witch to mentor more than two girls over her lifetime (she would graduate the second one in her sixties at the earliest). And many elect to train only a single designated successor.



                                                      Of course, basic control gets taught early on, and you can have a non-volatile girl just a year after her magic manifests. Yet, without further training she wouldn't contribute much to the coven, and it would be such a waste (especially for her mentor) to just cut training short there. And if an unclaimed girl comes of age for whatever reason (say, her designated mentor met an accident), then either someone free (and unwilling) has to teach her, or someone has to halt teaching their current apprentice to deal with a new menace.



                                                      Thus, it's not that boys are preferred. But there is a hard limit on how many girls are acceptable, and everyone else has to be boys. Assuming witches aren't too chaste, yet don't use contraception, this will be a natural consequence.






                                                      share|improve this answer






















                                                        up vote
                                                        4
                                                        down vote










                                                        up vote
                                                        4
                                                        down vote









                                                        Untrained witches are dangerous to everyone. And you can train only so many at once.



                                                        Every daughter of a witch has magic. But it is training that makes a witch out of a girl. Without training, magic is still there, but it is spontaneous, wild, bursting out at random. None wants to be around an untrained witch, as you have to maintain strongest protective charms at all time and still won't be completely sure if that would be enough.



                                                        On the other hand, the training involved is long and deeply personal. One full-fledged witch can train only one apprentice at a time. And the training up to the moment when she can be graduated and the next apprentice gets accepted takes two decades. Thus, it is extremely rare for a witch to mentor more than two girls over her lifetime (she would graduate the second one in her sixties at the earliest). And many elect to train only a single designated successor.



                                                        Of course, basic control gets taught early on, and you can have a non-volatile girl just a year after her magic manifests. Yet, without further training she wouldn't contribute much to the coven, and it would be such a waste (especially for her mentor) to just cut training short there. And if an unclaimed girl comes of age for whatever reason (say, her designated mentor met an accident), then either someone free (and unwilling) has to teach her, or someone has to halt teaching their current apprentice to deal with a new menace.



                                                        Thus, it's not that boys are preferred. But there is a hard limit on how many girls are acceptable, and everyone else has to be boys. Assuming witches aren't too chaste, yet don't use contraception, this will be a natural consequence.






                                                        share|improve this answer












                                                        Untrained witches are dangerous to everyone. And you can train only so many at once.



                                                        Every daughter of a witch has magic. But it is training that makes a witch out of a girl. Without training, magic is still there, but it is spontaneous, wild, bursting out at random. None wants to be around an untrained witch, as you have to maintain strongest protective charms at all time and still won't be completely sure if that would be enough.



                                                        On the other hand, the training involved is long and deeply personal. One full-fledged witch can train only one apprentice at a time. And the training up to the moment when she can be graduated and the next apprentice gets accepted takes two decades. Thus, it is extremely rare for a witch to mentor more than two girls over her lifetime (she would graduate the second one in her sixties at the earliest). And many elect to train only a single designated successor.



                                                        Of course, basic control gets taught early on, and you can have a non-volatile girl just a year after her magic manifests. Yet, without further training she wouldn't contribute much to the coven, and it would be such a waste (especially for her mentor) to just cut training short there. And if an unclaimed girl comes of age for whatever reason (say, her designated mentor met an accident), then either someone free (and unwilling) has to teach her, or someone has to halt teaching their current apprentice to deal with a new menace.



                                                        Thus, it's not that boys are preferred. But there is a hard limit on how many girls are acceptable, and everyone else has to be boys. Assuming witches aren't too chaste, yet don't use contraception, this will be a natural consequence.







                                                        share|improve this answer












                                                        share|improve this answer



                                                        share|improve this answer










                                                        answered Aug 30 at 9:42









                                                        Alice

                                                        6771511




                                                        6771511




















                                                            up vote
                                                            3
                                                            down vote














                                                            Eugenics.



                                                            In a medieval setup they will not have access to DNA sequencing or even knowledge about DNA but the witches know (by methodic observation and deep knowledge of nature) how the genetic inheritance work.



                                                            Sperm have more variety in genetic recombination than egg cells (spermatocytogenesis result in four cells while oogenesis produces only one) so the genetic content from sperm is valuable for eugenic process.



                                                            So, witches are told by their coven leader to give birth to males and with whom to mate, only chosen witches are allowed to give birth to females.




                                                            In the end, all of us know where this setup leads to...






                                                            share|improve this answer
























                                                              up vote
                                                              3
                                                              down vote














                                                              Eugenics.



                                                              In a medieval setup they will not have access to DNA sequencing or even knowledge about DNA but the witches know (by methodic observation and deep knowledge of nature) how the genetic inheritance work.



                                                              Sperm have more variety in genetic recombination than egg cells (spermatocytogenesis result in four cells while oogenesis produces only one) so the genetic content from sperm is valuable for eugenic process.



                                                              So, witches are told by their coven leader to give birth to males and with whom to mate, only chosen witches are allowed to give birth to females.




                                                              In the end, all of us know where this setup leads to...






                                                              share|improve this answer






















                                                                up vote
                                                                3
                                                                down vote










                                                                up vote
                                                                3
                                                                down vote










                                                                Eugenics.



                                                                In a medieval setup they will not have access to DNA sequencing or even knowledge about DNA but the witches know (by methodic observation and deep knowledge of nature) how the genetic inheritance work.



                                                                Sperm have more variety in genetic recombination than egg cells (spermatocytogenesis result in four cells while oogenesis produces only one) so the genetic content from sperm is valuable for eugenic process.



                                                                So, witches are told by their coven leader to give birth to males and with whom to mate, only chosen witches are allowed to give birth to females.




                                                                In the end, all of us know where this setup leads to...






                                                                share|improve this answer













                                                                Eugenics.



                                                                In a medieval setup they will not have access to DNA sequencing or even knowledge about DNA but the witches know (by methodic observation and deep knowledge of nature) how the genetic inheritance work.



                                                                Sperm have more variety in genetic recombination than egg cells (spermatocytogenesis result in four cells while oogenesis produces only one) so the genetic content from sperm is valuable for eugenic process.



                                                                So, witches are told by their coven leader to give birth to males and with whom to mate, only chosen witches are allowed to give birth to females.




                                                                In the end, all of us know where this setup leads to...







                                                                share|improve this answer












                                                                share|improve this answer



                                                                share|improve this answer










                                                                answered Aug 30 at 12:42









                                                                Paula_plus_plus

                                                                346311




                                                                346311




















                                                                    up vote
                                                                    2
                                                                    down vote













                                                                    Out of kindness



                                                                    Because there is somehow a great psychic/psychological burden to witchcraft that it is, in itself, some kind of great curse or scouring: the greatest curse the witch can craft being to induce witchcraft in another. This can be the case despite external benefits. There are many stories where supernatural gifts are a burden to the beholder.






                                                                    share|improve this answer
























                                                                      up vote
                                                                      2
                                                                      down vote













                                                                      Out of kindness



                                                                      Because there is somehow a great psychic/psychological burden to witchcraft that it is, in itself, some kind of great curse or scouring: the greatest curse the witch can craft being to induce witchcraft in another. This can be the case despite external benefits. There are many stories where supernatural gifts are a burden to the beholder.






                                                                      share|improve this answer






















                                                                        up vote
                                                                        2
                                                                        down vote










                                                                        up vote
                                                                        2
                                                                        down vote









                                                                        Out of kindness



                                                                        Because there is somehow a great psychic/psychological burden to witchcraft that it is, in itself, some kind of great curse or scouring: the greatest curse the witch can craft being to induce witchcraft in another. This can be the case despite external benefits. There are many stories where supernatural gifts are a burden to the beholder.






                                                                        share|improve this answer












                                                                        Out of kindness



                                                                        Because there is somehow a great psychic/psychological burden to witchcraft that it is, in itself, some kind of great curse or scouring: the greatest curse the witch can craft being to induce witchcraft in another. This can be the case despite external benefits. There are many stories where supernatural gifts are a burden to the beholder.







                                                                        share|improve this answer












                                                                        share|improve this answer



                                                                        share|improve this answer










                                                                        answered Aug 30 at 0:37









                                                                        Dannie

                                                                        1211




                                                                        1211




















                                                                            up vote
                                                                            2
                                                                            down vote













                                                                            It's a matriarchy, so women are elites that have all the power. Lots of men all compete to be the best man to produce the next generation of witches, so more men equals more choices.






                                                                            share|improve this answer


























                                                                              up vote
                                                                              2
                                                                              down vote













                                                                              It's a matriarchy, so women are elites that have all the power. Lots of men all compete to be the best man to produce the next generation of witches, so more men equals more choices.






                                                                              share|improve this answer
























                                                                                up vote
                                                                                2
                                                                                down vote










                                                                                up vote
                                                                                2
                                                                                down vote









                                                                                It's a matriarchy, so women are elites that have all the power. Lots of men all compete to be the best man to produce the next generation of witches, so more men equals more choices.






                                                                                share|improve this answer














                                                                                It's a matriarchy, so women are elites that have all the power. Lots of men all compete to be the best man to produce the next generation of witches, so more men equals more choices.







                                                                                share|improve this answer














                                                                                share|improve this answer



                                                                                share|improve this answer








                                                                                edited Aug 30 at 1:03









                                                                                Brythan

                                                                                19.5k74182




                                                                                19.5k74182










                                                                                answered Aug 29 at 21:44









                                                                                Clay Deitas

                                                                                3,490722




                                                                                3,490722




















                                                                                    up vote
                                                                                    2
                                                                                    down vote













                                                                                    Lineage



                                                                                    Men, though unable to use magic, inherit the greater party of parental power.
                                                                                    Thus boys will be used for arranged marriages against dowry.
                                                                                    This would allow the greatest witch families to easily perpetuate and increase their power from one generation to the next.
                                                                                    Introducing the notion of nobility and lineage.
                                                                                    In the same way arranged marriages will allow to enrich the family, to rise up the ranks of society.



                                                                                    Moreover, a line that has "lost" its power could rise by marrying a girl to a young boy of a famous lineage



                                                                                    In conclusion, to answer your question: it would be advantageous to have a single daughter with several brothers, since a boy could be sold married to other families. Boys married or not will not be a burden because they would find work (crafts, soldiers, finances, etc.)






                                                                                    share|improve this answer


























                                                                                      up vote
                                                                                      2
                                                                                      down vote













                                                                                      Lineage



                                                                                      Men, though unable to use magic, inherit the greater party of parental power.
                                                                                      Thus boys will be used for arranged marriages against dowry.
                                                                                      This would allow the greatest witch families to easily perpetuate and increase their power from one generation to the next.
                                                                                      Introducing the notion of nobility and lineage.
                                                                                      In the same way arranged marriages will allow to enrich the family, to rise up the ranks of society.



                                                                                      Moreover, a line that has "lost" its power could rise by marrying a girl to a young boy of a famous lineage



                                                                                      In conclusion, to answer your question: it would be advantageous to have a single daughter with several brothers, since a boy could be sold married to other families. Boys married or not will not be a burden because they would find work (crafts, soldiers, finances, etc.)






                                                                                      share|improve this answer
























                                                                                        up vote
                                                                                        2
                                                                                        down vote










                                                                                        up vote
                                                                                        2
                                                                                        down vote









                                                                                        Lineage



                                                                                        Men, though unable to use magic, inherit the greater party of parental power.
                                                                                        Thus boys will be used for arranged marriages against dowry.
                                                                                        This would allow the greatest witch families to easily perpetuate and increase their power from one generation to the next.
                                                                                        Introducing the notion of nobility and lineage.
                                                                                        In the same way arranged marriages will allow to enrich the family, to rise up the ranks of society.



                                                                                        Moreover, a line that has "lost" its power could rise by marrying a girl to a young boy of a famous lineage



                                                                                        In conclusion, to answer your question: it would be advantageous to have a single daughter with several brothers, since a boy could be sold married to other families. Boys married or not will not be a burden because they would find work (crafts, soldiers, finances, etc.)






                                                                                        share|improve this answer














                                                                                        Lineage



                                                                                        Men, though unable to use magic, inherit the greater party of parental power.
                                                                                        Thus boys will be used for arranged marriages against dowry.
                                                                                        This would allow the greatest witch families to easily perpetuate and increase their power from one generation to the next.
                                                                                        Introducing the notion of nobility and lineage.
                                                                                        In the same way arranged marriages will allow to enrich the family, to rise up the ranks of society.



                                                                                        Moreover, a line that has "lost" its power could rise by marrying a girl to a young boy of a famous lineage



                                                                                        In conclusion, to answer your question: it would be advantageous to have a single daughter with several brothers, since a boy could be sold married to other families. Boys married or not will not be a burden because they would find work (crafts, soldiers, finances, etc.)







                                                                                        share|improve this answer














                                                                                        share|improve this answer



                                                                                        share|improve this answer








                                                                                        edited Aug 30 at 9:24

























                                                                                        answered Aug 30 at 8:46









                                                                                        Quentin Cespedes

                                                                                        313




                                                                                        313




















                                                                                            up vote
                                                                                            2
                                                                                            down vote













                                                                                            A witch’s time is incredibly valuable. Every minute she’s not casting spells or performing rituals or experimenting with new magic is wasted.



                                                                                            So men do everything else. Not just the traditional “man’s work” chores like putting up the shelves, but also the traditional “women’s work” chores like cooking and cleaning, and the specialized work like raising the newts and harvesting their eyes, and dealing with the world outside the coven, and even jobs that we’d consider prestigious like engineering that in their world are still not worthy of a witch’s time.



                                                                                            It’s not about keeping the men down; in fact, “everyone knows” that men are better at cooking and cleaning, as well as math and singing, That isn’t actually true (any more than it’s true that women are better at sweeping and men are better at doctoring in our world), but everyone believes it any way (just as people did in our world until the last century or so).



                                                                                            The one thing the men can’t do is teach new witches, which is a very time-intensive task. Schooling can only do so much; magic is highly personal, and much of it has to be passed on directly from mother to daughter. If you have four daughters, that quadruples the time you have to spend teaching them, which is time as you have to take away from actual witching. (What happens if a mother dies? Her nearest female relative with no daughters is expected to adopt and raise the girl instead of having one of her own.)



                                                                                            But if you have one daughter and three sons, that’s three extra people that can be useful to the coven without needing to be trained by a witch, so you’re wasting a lot less time.



                                                                                            Why couldn’t there just be women who aren’t witches, so they can divide the labor between men and mundane women? Well, you could have a daughter and not raise her to be a witch, but it would be considered horribly uncouth. People would think you’re being unconscionably cruel to your daughter, not to mention depriving the community of a witch.



                                                                                            Maybe there are some cultures that did develop that way, but every major civilization today traces its descent to a handful of cultures (like the Greek, Hebrew, Indus Valley, and Yellow River cultures in our world) that don’t, so as far as everyone is concerned, this is the way things have always been. The different major civilizations can be different in major ways (e.g., westerners are polyandrous, centralians encourage male homosexuality, easters expect women to be serially monogamous and men compete to be a woman’s last baby-daddy, the one who gets to stay with her through menopause and into eventual retirement), but they all have no place for non-witch women in their sexual politics. The ones who did are thought of like the Philistines or Barbarians of our world.






                                                                                            share|improve this answer




















                                                                                            • This sounds like a self weakening argument to me. We don't make witches because they are too valuable
                                                                                              – Andrey
                                                                                              Aug 30 at 18:52










                                                                                            • @Andrey No, we don’t make more witches because they take too much training. That’s what the whole fourth paragraph is about.
                                                                                              – abarnert
                                                                                              Aug 30 at 19:29










                                                                                            • If it takes more than 1 witch working full time to train a witch you would have 0 witches. If taring witches is a positive outcome you would train as many as you can because you know they are extremely valuable.
                                                                                              – Andrey
                                                                                              Aug 30 at 20:02














                                                                                            up vote
                                                                                            2
                                                                                            down vote













                                                                                            A witch’s time is incredibly valuable. Every minute she’s not casting spells or performing rituals or experimenting with new magic is wasted.



                                                                                            So men do everything else. Not just the traditional “man’s work” chores like putting up the shelves, but also the traditional “women’s work” chores like cooking and cleaning, and the specialized work like raising the newts and harvesting their eyes, and dealing with the world outside the coven, and even jobs that we’d consider prestigious like engineering that in their world are still not worthy of a witch’s time.



                                                                                            It’s not about keeping the men down; in fact, “everyone knows” that men are better at cooking and cleaning, as well as math and singing, That isn’t actually true (any more than it’s true that women are better at sweeping and men are better at doctoring in our world), but everyone believes it any way (just as people did in our world until the last century or so).



                                                                                            The one thing the men can’t do is teach new witches, which is a very time-intensive task. Schooling can only do so much; magic is highly personal, and much of it has to be passed on directly from mother to daughter. If you have four daughters, that quadruples the time you have to spend teaching them, which is time as you have to take away from actual witching. (What happens if a mother dies? Her nearest female relative with no daughters is expected to adopt and raise the girl instead of having one of her own.)



                                                                                            But if you have one daughter and three sons, that’s three extra people that can be useful to the coven without needing to be trained by a witch, so you’re wasting a lot less time.



                                                                                            Why couldn’t there just be women who aren’t witches, so they can divide the labor between men and mundane women? Well, you could have a daughter and not raise her to be a witch, but it would be considered horribly uncouth. People would think you’re being unconscionably cruel to your daughter, not to mention depriving the community of a witch.



                                                                                            Maybe there are some cultures that did develop that way, but every major civilization today traces its descent to a handful of cultures (like the Greek, Hebrew, Indus Valley, and Yellow River cultures in our world) that don’t, so as far as everyone is concerned, this is the way things have always been. The different major civilizations can be different in major ways (e.g., westerners are polyandrous, centralians encourage male homosexuality, easters expect women to be serially monogamous and men compete to be a woman’s last baby-daddy, the one who gets to stay with her through menopause and into eventual retirement), but they all have no place for non-witch women in their sexual politics. The ones who did are thought of like the Philistines or Barbarians of our world.






                                                                                            share|improve this answer




















                                                                                            • This sounds like a self weakening argument to me. We don't make witches because they are too valuable
                                                                                              – Andrey
                                                                                              Aug 30 at 18:52










                                                                                            • @Andrey No, we don’t make more witches because they take too much training. That’s what the whole fourth paragraph is about.
                                                                                              – abarnert
                                                                                              Aug 30 at 19:29










                                                                                            • If it takes more than 1 witch working full time to train a witch you would have 0 witches. If taring witches is a positive outcome you would train as many as you can because you know they are extremely valuable.
                                                                                              – Andrey
                                                                                              Aug 30 at 20:02












                                                                                            up vote
                                                                                            2
                                                                                            down vote










                                                                                            up vote
                                                                                            2
                                                                                            down vote









                                                                                            A witch’s time is incredibly valuable. Every minute she’s not casting spells or performing rituals or experimenting with new magic is wasted.



                                                                                            So men do everything else. Not just the traditional “man’s work” chores like putting up the shelves, but also the traditional “women’s work” chores like cooking and cleaning, and the specialized work like raising the newts and harvesting their eyes, and dealing with the world outside the coven, and even jobs that we’d consider prestigious like engineering that in their world are still not worthy of a witch’s time.



                                                                                            It’s not about keeping the men down; in fact, “everyone knows” that men are better at cooking and cleaning, as well as math and singing, That isn’t actually true (any more than it’s true that women are better at sweeping and men are better at doctoring in our world), but everyone believes it any way (just as people did in our world until the last century or so).



                                                                                            The one thing the men can’t do is teach new witches, which is a very time-intensive task. Schooling can only do so much; magic is highly personal, and much of it has to be passed on directly from mother to daughter. If you have four daughters, that quadruples the time you have to spend teaching them, which is time as you have to take away from actual witching. (What happens if a mother dies? Her nearest female relative with no daughters is expected to adopt and raise the girl instead of having one of her own.)



                                                                                            But if you have one daughter and three sons, that’s three extra people that can be useful to the coven without needing to be trained by a witch, so you’re wasting a lot less time.



                                                                                            Why couldn’t there just be women who aren’t witches, so they can divide the labor between men and mundane women? Well, you could have a daughter and not raise her to be a witch, but it would be considered horribly uncouth. People would think you’re being unconscionably cruel to your daughter, not to mention depriving the community of a witch.



                                                                                            Maybe there are some cultures that did develop that way, but every major civilization today traces its descent to a handful of cultures (like the Greek, Hebrew, Indus Valley, and Yellow River cultures in our world) that don’t, so as far as everyone is concerned, this is the way things have always been. The different major civilizations can be different in major ways (e.g., westerners are polyandrous, centralians encourage male homosexuality, easters expect women to be serially monogamous and men compete to be a woman’s last baby-daddy, the one who gets to stay with her through menopause and into eventual retirement), but they all have no place for non-witch women in their sexual politics. The ones who did are thought of like the Philistines or Barbarians of our world.






                                                                                            share|improve this answer












                                                                                            A witch’s time is incredibly valuable. Every minute she’s not casting spells or performing rituals or experimenting with new magic is wasted.



                                                                                            So men do everything else. Not just the traditional “man’s work” chores like putting up the shelves, but also the traditional “women’s work” chores like cooking and cleaning, and the specialized work like raising the newts and harvesting their eyes, and dealing with the world outside the coven, and even jobs that we’d consider prestigious like engineering that in their world are still not worthy of a witch’s time.



                                                                                            It’s not about keeping the men down; in fact, “everyone knows” that men are better at cooking and cleaning, as well as math and singing, That isn’t actually true (any more than it’s true that women are better at sweeping and men are better at doctoring in our world), but everyone believes it any way (just as people did in our world until the last century or so).



                                                                                            The one thing the men can’t do is teach new witches, which is a very time-intensive task. Schooling can only do so much; magic is highly personal, and much of it has to be passed on directly from mother to daughter. If you have four daughters, that quadruples the time you have to spend teaching them, which is time as you have to take away from actual witching. (What happens if a mother dies? Her nearest female relative with no daughters is expected to adopt and raise the girl instead of having one of her own.)



                                                                                            But if you have one daughter and three sons, that’s three extra people that can be useful to the coven without needing to be trained by a witch, so you’re wasting a lot less time.



                                                                                            Why couldn’t there just be women who aren’t witches, so they can divide the labor between men and mundane women? Well, you could have a daughter and not raise her to be a witch, but it would be considered horribly uncouth. People would think you’re being unconscionably cruel to your daughter, not to mention depriving the community of a witch.



                                                                                            Maybe there are some cultures that did develop that way, but every major civilization today traces its descent to a handful of cultures (like the Greek, Hebrew, Indus Valley, and Yellow River cultures in our world) that don’t, so as far as everyone is concerned, this is the way things have always been. The different major civilizations can be different in major ways (e.g., westerners are polyandrous, centralians encourage male homosexuality, easters expect women to be serially monogamous and men compete to be a woman’s last baby-daddy, the one who gets to stay with her through menopause and into eventual retirement), but they all have no place for non-witch women in their sexual politics. The ones who did are thought of like the Philistines or Barbarians of our world.







                                                                                            share|improve this answer












                                                                                            share|improve this answer



                                                                                            share|improve this answer










                                                                                            answered Aug 30 at 15:56









                                                                                            abarnert

                                                                                            1,35913




                                                                                            1,35913











                                                                                            • This sounds like a self weakening argument to me. We don't make witches because they are too valuable
                                                                                              – Andrey
                                                                                              Aug 30 at 18:52










                                                                                            • @Andrey No, we don’t make more witches because they take too much training. That’s what the whole fourth paragraph is about.
                                                                                              – abarnert
                                                                                              Aug 30 at 19:29










                                                                                            • If it takes more than 1 witch working full time to train a witch you would have 0 witches. If taring witches is a positive outcome you would train as many as you can because you know they are extremely valuable.
                                                                                              – Andrey
                                                                                              Aug 30 at 20:02
















                                                                                            • This sounds like a self weakening argument to me. We don't make witches because they are too valuable
                                                                                              – Andrey
                                                                                              Aug 30 at 18:52










                                                                                            • @Andrey No, we don’t make more witches because they take too much training. That’s what the whole fourth paragraph is about.
                                                                                              – abarnert
                                                                                              Aug 30 at 19:29










                                                                                            • If it takes more than 1 witch working full time to train a witch you would have 0 witches. If taring witches is a positive outcome you would train as many as you can because you know they are extremely valuable.
                                                                                              – Andrey
                                                                                              Aug 30 at 20:02















                                                                                            This sounds like a self weakening argument to me. We don't make witches because they are too valuable
                                                                                            – Andrey
                                                                                            Aug 30 at 18:52




                                                                                            This sounds like a self weakening argument to me. We don't make witches because they are too valuable
                                                                                            – Andrey
                                                                                            Aug 30 at 18:52












                                                                                            @Andrey No, we don’t make more witches because they take too much training. That’s what the whole fourth paragraph is about.
                                                                                            – abarnert
                                                                                            Aug 30 at 19:29




                                                                                            @Andrey No, we don’t make more witches because they take too much training. That’s what the whole fourth paragraph is about.
                                                                                            – abarnert
                                                                                            Aug 30 at 19:29












                                                                                            If it takes more than 1 witch working full time to train a witch you would have 0 witches. If taring witches is a positive outcome you would train as many as you can because you know they are extremely valuable.
                                                                                            – Andrey
                                                                                            Aug 30 at 20:02




                                                                                            If it takes more than 1 witch working full time to train a witch you would have 0 witches. If taring witches is a positive outcome you would train as many as you can because you know they are extremely valuable.
                                                                                            – Andrey
                                                                                            Aug 30 at 20:02










                                                                                            up vote
                                                                                            1
                                                                                            down vote













                                                                                            While in theory only one man is needed to get offsprings for good large willage of females, probably (if virginity is not required) powerful one (females) would prefere to have at least choise from man (not only one god or bad) eventually have even each ones harem.






                                                                                            share|improve this answer
























                                                                                              up vote
                                                                                              1
                                                                                              down vote













                                                                                              While in theory only one man is needed to get offsprings for good large willage of females, probably (if virginity is not required) powerful one (females) would prefere to have at least choise from man (not only one god or bad) eventually have even each ones harem.






                                                                                              share|improve this answer






















                                                                                                up vote
                                                                                                1
                                                                                                down vote










                                                                                                up vote
                                                                                                1
                                                                                                down vote









                                                                                                While in theory only one man is needed to get offsprings for good large willage of females, probably (if virginity is not required) powerful one (females) would prefere to have at least choise from man (not only one god or bad) eventually have even each ones harem.






                                                                                                share|improve this answer












                                                                                                While in theory only one man is needed to get offsprings for good large willage of females, probably (if virginity is not required) powerful one (females) would prefere to have at least choise from man (not only one god or bad) eventually have even each ones harem.







                                                                                                share|improve this answer












                                                                                                share|improve this answer



                                                                                                share|improve this answer










                                                                                                answered Aug 29 at 19:22









                                                                                                gilhad

                                                                                                43124




                                                                                                43124




















                                                                                                    up vote
                                                                                                    1
                                                                                                    down vote













                                                                                                    The amount of mana available to use for magic is divided among the current witches. Too many accessing it's powers will diminish the powers of them all. Having more males ensures that the witches in power stay powerful. Have another female born when a witch is retiring (dying) or has died already in order to bring it back to balance.






                                                                                                    share|improve this answer


























                                                                                                      up vote
                                                                                                      1
                                                                                                      down vote













                                                                                                      The amount of mana available to use for magic is divided among the current witches. Too many accessing it's powers will diminish the powers of them all. Having more males ensures that the witches in power stay powerful. Have another female born when a witch is retiring (dying) or has died already in order to bring it back to balance.






                                                                                                      share|improve this answer
























                                                                                                        up vote
                                                                                                        1
                                                                                                        down vote










                                                                                                        up vote
                                                                                                        1
                                                                                                        down vote









                                                                                                        The amount of mana available to use for magic is divided among the current witches. Too many accessing it's powers will diminish the powers of them all. Having more males ensures that the witches in power stay powerful. Have another female born when a witch is retiring (dying) or has died already in order to bring it back to balance.






                                                                                                        share|improve this answer














                                                                                                        The amount of mana available to use for magic is divided among the current witches. Too many accessing it's powers will diminish the powers of them all. Having more males ensures that the witches in power stay powerful. Have another female born when a witch is retiring (dying) or has died already in order to bring it back to balance.







                                                                                                        share|improve this answer














                                                                                                        share|improve this answer



                                                                                                        share|improve this answer








                                                                                                        edited Aug 30 at 12:59

























                                                                                                        answered Aug 30 at 11:42









                                                                                                        HDPZ

                                                                                                        113




                                                                                                        113




















                                                                                                            up vote
                                                                                                            1
                                                                                                            down vote













                                                                                                            The society allows for witches to have multiple husbands, so you have lots of male children to marry them off to other covens in order to cement alliances.



                                                                                                            Marrying off daughters would weaken you, so use sons.



                                                                                                            Also, everybody has magic - they don't need any particular material good, so you can't "buy" allies the same way you otherwise could. But what's the one thing a sensible witch can't make herself? Men that are safe to have children with. So you trade those.



                                                                                                            As a final bonus, men can be pretty useful if you don't feel like casting spells to fetch water, build walls, and do other simple tasks.






                                                                                                            share|improve this answer
























                                                                                                              up vote
                                                                                                              1
                                                                                                              down vote













                                                                                                              The society allows for witches to have multiple husbands, so you have lots of male children to marry them off to other covens in order to cement alliances.



                                                                                                              Marrying off daughters would weaken you, so use sons.



                                                                                                              Also, everybody has magic - they don't need any particular material good, so you can't "buy" allies the same way you otherwise could. But what's the one thing a sensible witch can't make herself? Men that are safe to have children with. So you trade those.



                                                                                                              As a final bonus, men can be pretty useful if you don't feel like casting spells to fetch water, build walls, and do other simple tasks.






                                                                                                              share|improve this answer






















                                                                                                                up vote
                                                                                                                1
                                                                                                                down vote










                                                                                                                up vote
                                                                                                                1
                                                                                                                down vote









                                                                                                                The society allows for witches to have multiple husbands, so you have lots of male children to marry them off to other covens in order to cement alliances.



                                                                                                                Marrying off daughters would weaken you, so use sons.



                                                                                                                Also, everybody has magic - they don't need any particular material good, so you can't "buy" allies the same way you otherwise could. But what's the one thing a sensible witch can't make herself? Men that are safe to have children with. So you trade those.



                                                                                                                As a final bonus, men can be pretty useful if you don't feel like casting spells to fetch water, build walls, and do other simple tasks.






                                                                                                                share|improve this answer












                                                                                                                The society allows for witches to have multiple husbands, so you have lots of male children to marry them off to other covens in order to cement alliances.



                                                                                                                Marrying off daughters would weaken you, so use sons.



                                                                                                                Also, everybody has magic - they don't need any particular material good, so you can't "buy" allies the same way you otherwise could. But what's the one thing a sensible witch can't make herself? Men that are safe to have children with. So you trade those.



                                                                                                                As a final bonus, men can be pretty useful if you don't feel like casting spells to fetch water, build walls, and do other simple tasks.







                                                                                                                share|improve this answer












                                                                                                                share|improve this answer



                                                                                                                share|improve this answer










                                                                                                                answered Aug 30 at 14:50









                                                                                                                Jeutnarg

                                                                                                                2,603519




                                                                                                                2,603519




















                                                                                                                    up vote
                                                                                                                    1
                                                                                                                    down vote













                                                                                                                    Human sacrifice is required for more complicated spells. The women certainly would not reduce the magic from the gene pool by sacrificing females/witches. That leaves those mostly worthless men as fodder for their dark spells.






                                                                                                                    share|improve this answer
























                                                                                                                      up vote
                                                                                                                      1
                                                                                                                      down vote













                                                                                                                      Human sacrifice is required for more complicated spells. The women certainly would not reduce the magic from the gene pool by sacrificing females/witches. That leaves those mostly worthless men as fodder for their dark spells.






                                                                                                                      share|improve this answer






















                                                                                                                        up vote
                                                                                                                        1
                                                                                                                        down vote










                                                                                                                        up vote
                                                                                                                        1
                                                                                                                        down vote









                                                                                                                        Human sacrifice is required for more complicated spells. The women certainly would not reduce the magic from the gene pool by sacrificing females/witches. That leaves those mostly worthless men as fodder for their dark spells.






                                                                                                                        share|improve this answer












                                                                                                                        Human sacrifice is required for more complicated spells. The women certainly would not reduce the magic from the gene pool by sacrificing females/witches. That leaves those mostly worthless men as fodder for their dark spells.







                                                                                                                        share|improve this answer












                                                                                                                        share|improve this answer



                                                                                                                        share|improve this answer










                                                                                                                        answered Aug 30 at 15:35









                                                                                                                        UnhandledExcepSean

                                                                                                                        1113




                                                                                                                        1113




















                                                                                                                            up vote
                                                                                                                            1
                                                                                                                            down vote













                                                                                                                            The Iroquois Confederacy had a rather unique gender role system. Women had more political power, but men were vital to society as well. Generally, Men were used in matters of foreign affairs - Diplomacy and War. When the confederation formed, Men were sent to the confederation council (the Sachem or Grand Sachem) to represent the local tribe's female exclusive council. The local councils had more power technically, because they had the right to recall their representative (called Horn Knocking due to ceremonial importance of deer horns in the headdress among the male leadership). Both men and women councils had to reach a 75% consensus for a decision to pass.



                                                                                                                            In the home front, the women owned the property and it was passed down matrilinearly and men were adopted into the family of their wives and was considered on the level of his wife's brothers in the household. Men were also exclusively warriors but only women had the ability to declare the war. This lead to a rather unique war philosophy called "Mourning Wars" which sought captives to replace deceased members of the family and tribe (often the war dead from a prievious attack or raid). Thus, the goal of wars for the Confederacy was to capture as many people and bring them back alive to be adopted into the tribe, not territorial gains. Thus slaughter of the enemy was seen as a bad tactic as it decreased the success of the war, and the war could be a failure if the number of war dead on their own side was not offset by the captives taken, so the Iroquois were quick to retreat.



                                                                                                                            Those that did not go to war were declared cowards by the clan mother and it was taboo for women to marry men declared as such. Thus, men went to war to prove they were worth of marriage. Typically, a male War Chief was declared who would have authority on the battle field but not much authority at home.



                                                                                                                            All of this existed due to the creation myth, which held that women were stewards of the land and must be respected in their decisions. Men traditionally hunted game while women tended to the farm and the land the clans controlled, and thus had greater importance. Thus their ability to name leaders of the tribes and clans. While men would be the face of the Confederacy to outsiders, women were often the power behind the throne. This could translate into a matrilinear society where the magic of women was such that it was better for them to remain close to the home and not participate in the day to day management of the government by traveling to meeting bodies or court favor from the courts, but the men worked for them, not the reverse, and if they didn't like the outcome, they could remove the offender from office.



                                                                                                                            (It should be pointed out that Iroquois aren't considered a Matriarchal society... both Genders had political power and were quite important in governing the nation.).






                                                                                                                            share|improve this answer
























                                                                                                                              up vote
                                                                                                                              1
                                                                                                                              down vote













                                                                                                                              The Iroquois Confederacy had a rather unique gender role system. Women had more political power, but men were vital to society as well. Generally, Men were used in matters of foreign affairs - Diplomacy and War. When the confederation formed, Men were sent to the confederation council (the Sachem or Grand Sachem) to represent the local tribe's female exclusive council. The local councils had more power technically, because they had the right to recall their representative (called Horn Knocking due to ceremonial importance of deer horns in the headdress among the male leadership). Both men and women councils had to reach a 75% consensus for a decision to pass.



                                                                                                                              In the home front, the women owned the property and it was passed down matrilinearly and men were adopted into the family of their wives and was considered on the level of his wife's brothers in the household. Men were also exclusively warriors but only women had the ability to declare the war. This lead to a rather unique war philosophy called "Mourning Wars" which sought captives to replace deceased members of the family and tribe (often the war dead from a prievious attack or raid). Thus, the goal of wars for the Confederacy was to capture as many people and bring them back alive to be adopted into the tribe, not territorial gains. Thus slaughter of the enemy was seen as a bad tactic as it decreased the success of the war, and the war could be a failure if the number of war dead on their own side was not offset by the captives taken, so the Iroquois were quick to retreat.



                                                                                                                              Those that did not go to war were declared cowards by the clan mother and it was taboo for women to marry men declared as such. Thus, men went to war to prove they were worth of marriage. Typically, a male War Chief was declared who would have authority on the battle field but not much authority at home.



                                                                                                                              All of this existed due to the creation myth, which held that women were stewards of the land and must be respected in their decisions. Men traditionally hunted game while women tended to the farm and the land the clans controlled, and thus had greater importance. Thus their ability to name leaders of the tribes and clans. While men would be the face of the Confederacy to outsiders, women were often the power behind the throne. This could translate into a matrilinear society where the magic of women was such that it was better for them to remain close to the home and not participate in the day to day management of the government by traveling to meeting bodies or court favor from the courts, but the men worked for them, not the reverse, and if they didn't like the outcome, they could remove the offender from office.



                                                                                                                              (It should be pointed out that Iroquois aren't considered a Matriarchal society... both Genders had political power and were quite important in governing the nation.).






                                                                                                                              share|improve this answer






















                                                                                                                                up vote
                                                                                                                                1
                                                                                                                                down vote










                                                                                                                                up vote
                                                                                                                                1
                                                                                                                                down vote









                                                                                                                                The Iroquois Confederacy had a rather unique gender role system. Women had more political power, but men were vital to society as well. Generally, Men were used in matters of foreign affairs - Diplomacy and War. When the confederation formed, Men were sent to the confederation council (the Sachem or Grand Sachem) to represent the local tribe's female exclusive council. The local councils had more power technically, because they had the right to recall their representative (called Horn Knocking due to ceremonial importance of deer horns in the headdress among the male leadership). Both men and women councils had to reach a 75% consensus for a decision to pass.



                                                                                                                                In the home front, the women owned the property and it was passed down matrilinearly and men were adopted into the family of their wives and was considered on the level of his wife's brothers in the household. Men were also exclusively warriors but only women had the ability to declare the war. This lead to a rather unique war philosophy called "Mourning Wars" which sought captives to replace deceased members of the family and tribe (often the war dead from a prievious attack or raid). Thus, the goal of wars for the Confederacy was to capture as many people and bring them back alive to be adopted into the tribe, not territorial gains. Thus slaughter of the enemy was seen as a bad tactic as it decreased the success of the war, and the war could be a failure if the number of war dead on their own side was not offset by the captives taken, so the Iroquois were quick to retreat.



                                                                                                                                Those that did not go to war were declared cowards by the clan mother and it was taboo for women to marry men declared as such. Thus, men went to war to prove they were worth of marriage. Typically, a male War Chief was declared who would have authority on the battle field but not much authority at home.



                                                                                                                                All of this existed due to the creation myth, which held that women were stewards of the land and must be respected in their decisions. Men traditionally hunted game while women tended to the farm and the land the clans controlled, and thus had greater importance. Thus their ability to name leaders of the tribes and clans. While men would be the face of the Confederacy to outsiders, women were often the power behind the throne. This could translate into a matrilinear society where the magic of women was such that it was better for them to remain close to the home and not participate in the day to day management of the government by traveling to meeting bodies or court favor from the courts, but the men worked for them, not the reverse, and if they didn't like the outcome, they could remove the offender from office.



                                                                                                                                (It should be pointed out that Iroquois aren't considered a Matriarchal society... both Genders had political power and were quite important in governing the nation.).






                                                                                                                                share|improve this answer












                                                                                                                                The Iroquois Confederacy had a rather unique gender role system. Women had more political power, but men were vital to society as well. Generally, Men were used in matters of foreign affairs - Diplomacy and War. When the confederation formed, Men were sent to the confederation council (the Sachem or Grand Sachem) to represent the local tribe's female exclusive council. The local councils had more power technically, because they had the right to recall their representative (called Horn Knocking due to ceremonial importance of deer horns in the headdress among the male leadership). Both men and women councils had to reach a 75% consensus for a decision to pass.



                                                                                                                                In the home front, the women owned the property and it was passed down matrilinearly and men were adopted into the family of their wives and was considered on the level of his wife's brothers in the household. Men were also exclusively warriors but only women had the ability to declare the war. This lead to a rather unique war philosophy called "Mourning Wars" which sought captives to replace deceased members of the family and tribe (often the war dead from a prievious attack or raid). Thus, the goal of wars for the Confederacy was to capture as many people and bring them back alive to be adopted into the tribe, not territorial gains. Thus slaughter of the enemy was seen as a bad tactic as it decreased the success of the war, and the war could be a failure if the number of war dead on their own side was not offset by the captives taken, so the Iroquois were quick to retreat.



                                                                                                                                Those that did not go to war were declared cowards by the clan mother and it was taboo for women to marry men declared as such. Thus, men went to war to prove they were worth of marriage. Typically, a male War Chief was declared who would have authority on the battle field but not much authority at home.



                                                                                                                                All of this existed due to the creation myth, which held that women were stewards of the land and must be respected in their decisions. Men traditionally hunted game while women tended to the farm and the land the clans controlled, and thus had greater importance. Thus their ability to name leaders of the tribes and clans. While men would be the face of the Confederacy to outsiders, women were often the power behind the throne. This could translate into a matrilinear society where the magic of women was such that it was better for them to remain close to the home and not participate in the day to day management of the government by traveling to meeting bodies or court favor from the courts, but the men worked for them, not the reverse, and if they didn't like the outcome, they could remove the offender from office.



                                                                                                                                (It should be pointed out that Iroquois aren't considered a Matriarchal society... both Genders had political power and were quite important in governing the nation.).







                                                                                                                                share|improve this answer












                                                                                                                                share|improve this answer



                                                                                                                                share|improve this answer










                                                                                                                                answered Aug 31 at 12:32









                                                                                                                                hszmv

                                                                                                                                3,606311




                                                                                                                                3,606311




















                                                                                                                                    up vote
                                                                                                                                    0
                                                                                                                                    down vote













                                                                                                                                    Marriage as Business



                                                                                                                                    In the first minutes of Netflix's Explained chapter about monogamy, the narrator states that some historians believe that marriage was invented as a way to have a "gain" for your family - a means to join forces with another "clan". They give several examples as to how marriage was used to join powerful families and mantain control over the ages (Cleopatra and the romans).



                                                                                                                                    The reason for men to exist could be linked to that. If powerful witches have a high social standing, it would be only natural that a certain witch would have a male son to join two powerful "witch houses".



                                                                                                                                    Women are Rare



                                                                                                                                    Women already play a central role in society without magic powers. They have children and thus, are the only ones capable of perpetuating society.



                                                                                                                                    But what if having a female child was really hard?



                                                                                                                                    Since they manage to control mana and be very powerful, there could be some magical BS that made women really hard to come by - to the point that witches would actually rely on magic to try and shift those odds for a better chance for a girl to be born.



                                                                                                                                    Some people have already said here in here that maybe a newborn witch draws a portion of the mother's power. Maybe that's true and, therefore, only the most powerful witches could have a chance to give birth to a girl - and that would certainly take a toll on the mother.



                                                                                                                                    Maybe it could even be a death sentence to try and give birth to a woman!



                                                                                                                                    Seventh Son of a Seventh Son



                                                                                                                                    I don't know where this saying originated, but it has something to do with great power.



                                                                                                                                    Suppose all witches have to be the seventh children - after an unbroken line of six male sons - of a mother who, in turn, is a seventh children herself.



                                                                                                                                    This would fairly explain why are there so much more men than women and also why they would rely on magic to have a lot of male children before a woman is born.



                                                                                                                                    An Unorthodox Magic Component



                                                                                                                                    Having a male children could be a magic component for some desired spell - like eternal life.



                                                                                                                                    Suppose there's a ritual that gives you 10 more years to live. Witches would certainly cast this spell multiple times through the years in order to live for centuries.



                                                                                                                                    Of course it would have to be a high level spell and, therefore, require a lot of time and resources - and a male children could be one of these resources. In this question, I tried to make a silly game where people would come up with crazy magic components for spells. For this particular spell, one of the main components could be "being pregnant with a boy" or "giving birth to a boy".






                                                                                                                                    share|improve this answer




















                                                                                                                                    • I like your Seventh Child option, it has roots in a lot of common lore in fiction as well as folklore (for a modern example: In the Harry Potter books, Ginny Weasley is the only daughter and seventh child of the Weasley Family) Seems like an elegant answer because it necessitates a 6:1 ratio of men to women just to keep up the numbers of witches in the community, and a witch would only likely produce one other witch in her life..
                                                                                                                                      – Ruadhan
                                                                                                                                      Aug 31 at 8:29














                                                                                                                                    up vote
                                                                                                                                    0
                                                                                                                                    down vote













                                                                                                                                    Marriage as Business



                                                                                                                                    In the first minutes of Netflix's Explained chapter about monogamy, the narrator states that some historians believe that marriage was invented as a way to have a "gain" for your family - a means to join forces with another "clan". They give several examples as to how marriage was used to join powerful families and mantain control over the ages (Cleopatra and the romans).



                                                                                                                                    The reason for men to exist could be linked to that. If powerful witches have a high social standing, it would be only natural that a certain witch would have a male son to join two powerful "witch houses".



                                                                                                                                    Women are Rare



                                                                                                                                    Women already play a central role in society without magic powers. They have children and thus, are the only ones capable of perpetuating society.



                                                                                                                                    But what if having a female child was really hard?



                                                                                                                                    Since they manage to control mana and be very powerful, there could be some magical BS that made women really hard to come by - to the point that witches would actually rely on magic to try and shift those odds for a better chance for a girl to be born.



                                                                                                                                    Some people have already said here in here that maybe a newborn witch draws a portion of the mother's power. Maybe that's true and, therefore, only the most powerful witches could have a chance to give birth to a girl - and that would certainly take a toll on the mother.



                                                                                                                                    Maybe it could even be a death sentence to try and give birth to a woman!



                                                                                                                                    Seventh Son of a Seventh Son



                                                                                                                                    I don't know where this saying originated, but it has something to do with great power.



                                                                                                                                    Suppose all witches have to be the seventh children - after an unbroken line of six male sons - of a mother who, in turn, is a seventh children herself.



                                                                                                                                    This would fairly explain why are there so much more men than women and also why they would rely on magic to have a lot of male children before a woman is born.



                                                                                                                                    An Unorthodox Magic Component



                                                                                                                                    Having a male children could be a magic component for some desired spell - like eternal life.



                                                                                                                                    Suppose there's a ritual that gives you 10 more years to live. Witches would certainly cast this spell multiple times through the years in order to live for centuries.



                                                                                                                                    Of course it would have to be a high level spell and, therefore, require a lot of time and resources - and a male children could be one of these resources. In this question, I tried to make a silly game where people would come up with crazy magic components for spells. For this particular spell, one of the main components could be "being pregnant with a boy" or "giving birth to a boy".






                                                                                                                                    share|improve this answer




















                                                                                                                                    • I like your Seventh Child option, it has roots in a lot of common lore in fiction as well as folklore (for a modern example: In the Harry Potter books, Ginny Weasley is the only daughter and seventh child of the Weasley Family) Seems like an elegant answer because it necessitates a 6:1 ratio of men to women just to keep up the numbers of witches in the community, and a witch would only likely produce one other witch in her life..
                                                                                                                                      – Ruadhan
                                                                                                                                      Aug 31 at 8:29












                                                                                                                                    up vote
                                                                                                                                    0
                                                                                                                                    down vote










                                                                                                                                    up vote
                                                                                                                                    0
                                                                                                                                    down vote









                                                                                                                                    Marriage as Business



                                                                                                                                    In the first minutes of Netflix's Explained chapter about monogamy, the narrator states that some historians believe that marriage was invented as a way to have a "gain" for your family - a means to join forces with another "clan". They give several examples as to how marriage was used to join powerful families and mantain control over the ages (Cleopatra and the romans).



                                                                                                                                    The reason for men to exist could be linked to that. If powerful witches have a high social standing, it would be only natural that a certain witch would have a male son to join two powerful "witch houses".



                                                                                                                                    Women are Rare



                                                                                                                                    Women already play a central role in society without magic powers. They have children and thus, are the only ones capable of perpetuating society.



                                                                                                                                    But what if having a female child was really hard?



                                                                                                                                    Since they manage to control mana and be very powerful, there could be some magical BS that made women really hard to come by - to the point that witches would actually rely on magic to try and shift those odds for a better chance for a girl to be born.



                                                                                                                                    Some people have already said here in here that maybe a newborn witch draws a portion of the mother's power. Maybe that's true and, therefore, only the most powerful witches could have a chance to give birth to a girl - and that would certainly take a toll on the mother.



                                                                                                                                    Maybe it could even be a death sentence to try and give birth to a woman!



                                                                                                                                    Seventh Son of a Seventh Son



                                                                                                                                    I don't know where this saying originated, but it has something to do with great power.



                                                                                                                                    Suppose all witches have to be the seventh children - after an unbroken line of six male sons - of a mother who, in turn, is a seventh children herself.



                                                                                                                                    This would fairly explain why are there so much more men than women and also why they would rely on magic to have a lot of male children before a woman is born.



                                                                                                                                    An Unorthodox Magic Component



                                                                                                                                    Having a male children could be a magic component for some desired spell - like eternal life.



                                                                                                                                    Suppose there's a ritual that gives you 10 more years to live. Witches would certainly cast this spell multiple times through the years in order to live for centuries.



                                                                                                                                    Of course it would have to be a high level spell and, therefore, require a lot of time and resources - and a male children could be one of these resources. In this question, I tried to make a silly game where people would come up with crazy magic components for spells. For this particular spell, one of the main components could be "being pregnant with a boy" or "giving birth to a boy".






                                                                                                                                    share|improve this answer












                                                                                                                                    Marriage as Business



                                                                                                                                    In the first minutes of Netflix's Explained chapter about monogamy, the narrator states that some historians believe that marriage was invented as a way to have a "gain" for your family - a means to join forces with another "clan". They give several examples as to how marriage was used to join powerful families and mantain control over the ages (Cleopatra and the romans).



                                                                                                                                    The reason for men to exist could be linked to that. If powerful witches have a high social standing, it would be only natural that a certain witch would have a male son to join two powerful "witch houses".



                                                                                                                                    Women are Rare



                                                                                                                                    Women already play a central role in society without magic powers. They have children and thus, are the only ones capable of perpetuating society.



                                                                                                                                    But what if having a female child was really hard?



                                                                                                                                    Since they manage to control mana and be very powerful, there could be some magical BS that made women really hard to come by - to the point that witches would actually rely on magic to try and shift those odds for a better chance for a girl to be born.



                                                                                                                                    Some people have already said here in here that maybe a newborn witch draws a portion of the mother's power. Maybe that's true and, therefore, only the most powerful witches could have a chance to give birth to a girl - and that would certainly take a toll on the mother.



                                                                                                                                    Maybe it could even be a death sentence to try and give birth to a woman!



                                                                                                                                    Seventh Son of a Seventh Son



                                                                                                                                    I don't know where this saying originated, but it has something to do with great power.



                                                                                                                                    Suppose all witches have to be the seventh children - after an unbroken line of six male sons - of a mother who, in turn, is a seventh children herself.



                                                                                                                                    This would fairly explain why are there so much more men than women and also why they would rely on magic to have a lot of male children before a woman is born.



                                                                                                                                    An Unorthodox Magic Component



                                                                                                                                    Having a male children could be a magic component for some desired spell - like eternal life.



                                                                                                                                    Suppose there's a ritual that gives you 10 more years to live. Witches would certainly cast this spell multiple times through the years in order to live for centuries.



                                                                                                                                    Of course it would have to be a high level spell and, therefore, require a lot of time and resources - and a male children could be one of these resources. In this question, I tried to make a silly game where people would come up with crazy magic components for spells. For this particular spell, one of the main components could be "being pregnant with a boy" or "giving birth to a boy".







                                                                                                                                    share|improve this answer












                                                                                                                                    share|improve this answer



                                                                                                                                    share|improve this answer










                                                                                                                                    answered Aug 30 at 12:48









                                                                                                                                    Magus

                                                                                                                                    742517




                                                                                                                                    742517











                                                                                                                                    • I like your Seventh Child option, it has roots in a lot of common lore in fiction as well as folklore (for a modern example: In the Harry Potter books, Ginny Weasley is the only daughter and seventh child of the Weasley Family) Seems like an elegant answer because it necessitates a 6:1 ratio of men to women just to keep up the numbers of witches in the community, and a witch would only likely produce one other witch in her life..
                                                                                                                                      – Ruadhan
                                                                                                                                      Aug 31 at 8:29
















                                                                                                                                    • I like your Seventh Child option, it has roots in a lot of common lore in fiction as well as folklore (for a modern example: In the Harry Potter books, Ginny Weasley is the only daughter and seventh child of the Weasley Family) Seems like an elegant answer because it necessitates a 6:1 ratio of men to women just to keep up the numbers of witches in the community, and a witch would only likely produce one other witch in her life..
                                                                                                                                      – Ruadhan
                                                                                                                                      Aug 31 at 8:29















                                                                                                                                    I like your Seventh Child option, it has roots in a lot of common lore in fiction as well as folklore (for a modern example: In the Harry Potter books, Ginny Weasley is the only daughter and seventh child of the Weasley Family) Seems like an elegant answer because it necessitates a 6:1 ratio of men to women just to keep up the numbers of witches in the community, and a witch would only likely produce one other witch in her life..
                                                                                                                                    – Ruadhan
                                                                                                                                    Aug 31 at 8:29




                                                                                                                                    I like your Seventh Child option, it has roots in a lot of common lore in fiction as well as folklore (for a modern example: In the Harry Potter books, Ginny Weasley is the only daughter and seventh child of the Weasley Family) Seems like an elegant answer because it necessitates a 6:1 ratio of men to women just to keep up the numbers of witches in the community, and a witch would only likely produce one other witch in her life..
                                                                                                                                    – Ruadhan
                                                                                                                                    Aug 31 at 8:29










                                                                                                                                    up vote
                                                                                                                                    0
                                                                                                                                    down vote













                                                                                                                                    Dowry, which actually caused a gender imbalance in China.



                                                                                                                                    Men are expected to perform all of the non-magic chores. This includes gathering things needed by the household, feeding the family members, cleaning the home, and taking care of children and the elderly. Women can therefore focus their efforts on their magic. Although the practice may seem outdated to us, everyone does benefit by it, and it is the norm of that culture.



                                                                                                                                    When a couple marries, the husband is expected to care for the wife's aging parents. This means the husband is no longer available to care for his own parents. At the time of marriage, the wife's parents compensate the husband's parents by paying a dowry. The practice is very common in various real-world cultures, although the roles of gender are usually reversed.



                                                                                                                                    Something is causing couples to have fewer children. In China it was the One Child policy, but you can make up any reason you want. Supply and demand causes the cost of dowries to radically increase, which favors couples with sons and penalizes couples with daughters. Couples therefore choose to have more sons than daughters.



                                                                                                                                    This actually happened in China, which now has 30 million more males than females.






                                                                                                                                    share|improve this answer
























                                                                                                                                      up vote
                                                                                                                                      0
                                                                                                                                      down vote













                                                                                                                                      Dowry, which actually caused a gender imbalance in China.



                                                                                                                                      Men are expected to perform all of the non-magic chores. This includes gathering things needed by the household, feeding the family members, cleaning the home, and taking care of children and the elderly. Women can therefore focus their efforts on their magic. Although the practice may seem outdated to us, everyone does benefit by it, and it is the norm of that culture.



                                                                                                                                      When a couple marries, the husband is expected to care for the wife's aging parents. This means the husband is no longer available to care for his own parents. At the time of marriage, the wife's parents compensate the husband's parents by paying a dowry. The practice is very common in various real-world cultures, although the roles of gender are usually reversed.



                                                                                                                                      Something is causing couples to have fewer children. In China it was the One Child policy, but you can make up any reason you want. Supply and demand causes the cost of dowries to radically increase, which favors couples with sons and penalizes couples with daughters. Couples therefore choose to have more sons than daughters.



                                                                                                                                      This actually happened in China, which now has 30 million more males than females.






                                                                                                                                      share|improve this answer






















                                                                                                                                        up vote
                                                                                                                                        0
                                                                                                                                        down vote










                                                                                                                                        up vote
                                                                                                                                        0
                                                                                                                                        down vote









                                                                                                                                        Dowry, which actually caused a gender imbalance in China.



                                                                                                                                        Men are expected to perform all of the non-magic chores. This includes gathering things needed by the household, feeding the family members, cleaning the home, and taking care of children and the elderly. Women can therefore focus their efforts on their magic. Although the practice may seem outdated to us, everyone does benefit by it, and it is the norm of that culture.



                                                                                                                                        When a couple marries, the husband is expected to care for the wife's aging parents. This means the husband is no longer available to care for his own parents. At the time of marriage, the wife's parents compensate the husband's parents by paying a dowry. The practice is very common in various real-world cultures, although the roles of gender are usually reversed.



                                                                                                                                        Something is causing couples to have fewer children. In China it was the One Child policy, but you can make up any reason you want. Supply and demand causes the cost of dowries to radically increase, which favors couples with sons and penalizes couples with daughters. Couples therefore choose to have more sons than daughters.



                                                                                                                                        This actually happened in China, which now has 30 million more males than females.






                                                                                                                                        share|improve this answer












                                                                                                                                        Dowry, which actually caused a gender imbalance in China.



                                                                                                                                        Men are expected to perform all of the non-magic chores. This includes gathering things needed by the household, feeding the family members, cleaning the home, and taking care of children and the elderly. Women can therefore focus their efforts on their magic. Although the practice may seem outdated to us, everyone does benefit by it, and it is the norm of that culture.



                                                                                                                                        When a couple marries, the husband is expected to care for the wife's aging parents. This means the husband is no longer available to care for his own parents. At the time of marriage, the wife's parents compensate the husband's parents by paying a dowry. The practice is very common in various real-world cultures, although the roles of gender are usually reversed.



                                                                                                                                        Something is causing couples to have fewer children. In China it was the One Child policy, but you can make up any reason you want. Supply and demand causes the cost of dowries to radically increase, which favors couples with sons and penalizes couples with daughters. Couples therefore choose to have more sons than daughters.



                                                                                                                                        This actually happened in China, which now has 30 million more males than females.







                                                                                                                                        share|improve this answer












                                                                                                                                        share|improve this answer



                                                                                                                                        share|improve this answer










                                                                                                                                        answered Aug 30 at 13:38









                                                                                                                                        Dr Sheldon

                                                                                                                                        36719




                                                                                                                                        36719




















                                                                                                                                            up vote
                                                                                                                                            0
                                                                                                                                            down vote













                                                                                                                                            There could be a number of reasons,



                                                                                                                                            1. Hierarchical Limits: If only women can practice witchcraft then daughters = power. If the covens live in a society, then they are most likely organized and their power is restricted. The most powerful covens are allowed to have the most daughters, lesser covens fewer. The main question with this though is why would they get pregnant if they were 'full' on the number of daughters they could have. Which leads to the next possible reason.


                                                                                                                                            2. Maximizing Potential Members: So let's assume that the coven can only support so many members, either from the above scenario, or for some other reason such as available magic or resources. Logically they would want the best and the brightest. Let's suppose The coven can support 9 members but only has 8. Each baby born is examined, only daughters with the highest potential are carried to term, the rest become boys. This leads to an excess of male children.


                                                                                                                                            3. Breeding Stock: In this world only women can use witchcraft, but what if the potential to use witchcraft is mostly inherited from the male side? In this scenario then some males might be extremely valuable, and some well, might be more expendable.


                                                                                                                                            4. Cannon Fodder: Every army needs foot soldiers. Though from the description of the world Covens would already be on top. Perhaps as muscle to keep the non coven people in line and handle mundane tasks?






                                                                                                                                            share|improve this answer
























                                                                                                                                              up vote
                                                                                                                                              0
                                                                                                                                              down vote













                                                                                                                                              There could be a number of reasons,



                                                                                                                                              1. Hierarchical Limits: If only women can practice witchcraft then daughters = power. If the covens live in a society, then they are most likely organized and their power is restricted. The most powerful covens are allowed to have the most daughters, lesser covens fewer. The main question with this though is why would they get pregnant if they were 'full' on the number of daughters they could have. Which leads to the next possible reason.


                                                                                                                                              2. Maximizing Potential Members: So let's assume that the coven can only support so many members, either from the above scenario, or for some other reason such as available magic or resources. Logically they would want the best and the brightest. Let's suppose The coven can support 9 members but only has 8. Each baby born is examined, only daughters with the highest potential are carried to term, the rest become boys. This leads to an excess of male children.


                                                                                                                                              3. Breeding Stock: In this world only women can use witchcraft, but what if the potential to use witchcraft is mostly inherited from the male side? In this scenario then some males might be extremely valuable, and some well, might be more expendable.


                                                                                                                                              4. Cannon Fodder: Every army needs foot soldiers. Though from the description of the world Covens would already be on top. Perhaps as muscle to keep the non coven people in line and handle mundane tasks?






                                                                                                                                              share|improve this answer






















                                                                                                                                                up vote
                                                                                                                                                0
                                                                                                                                                down vote










                                                                                                                                                up vote
                                                                                                                                                0
                                                                                                                                                down vote









                                                                                                                                                There could be a number of reasons,



                                                                                                                                                1. Hierarchical Limits: If only women can practice witchcraft then daughters = power. If the covens live in a society, then they are most likely organized and their power is restricted. The most powerful covens are allowed to have the most daughters, lesser covens fewer. The main question with this though is why would they get pregnant if they were 'full' on the number of daughters they could have. Which leads to the next possible reason.


                                                                                                                                                2. Maximizing Potential Members: So let's assume that the coven can only support so many members, either from the above scenario, or for some other reason such as available magic or resources. Logically they would want the best and the brightest. Let's suppose The coven can support 9 members but only has 8. Each baby born is examined, only daughters with the highest potential are carried to term, the rest become boys. This leads to an excess of male children.


                                                                                                                                                3. Breeding Stock: In this world only women can use witchcraft, but what if the potential to use witchcraft is mostly inherited from the male side? In this scenario then some males might be extremely valuable, and some well, might be more expendable.


                                                                                                                                                4. Cannon Fodder: Every army needs foot soldiers. Though from the description of the world Covens would already be on top. Perhaps as muscle to keep the non coven people in line and handle mundane tasks?






                                                                                                                                                share|improve this answer












                                                                                                                                                There could be a number of reasons,



                                                                                                                                                1. Hierarchical Limits: If only women can practice witchcraft then daughters = power. If the covens live in a society, then they are most likely organized and their power is restricted. The most powerful covens are allowed to have the most daughters, lesser covens fewer. The main question with this though is why would they get pregnant if they were 'full' on the number of daughters they could have. Which leads to the next possible reason.


                                                                                                                                                2. Maximizing Potential Members: So let's assume that the coven can only support so many members, either from the above scenario, or for some other reason such as available magic or resources. Logically they would want the best and the brightest. Let's suppose The coven can support 9 members but only has 8. Each baby born is examined, only daughters with the highest potential are carried to term, the rest become boys. This leads to an excess of male children.


                                                                                                                                                3. Breeding Stock: In this world only women can use witchcraft, but what if the potential to use witchcraft is mostly inherited from the male side? In this scenario then some males might be extremely valuable, and some well, might be more expendable.


                                                                                                                                                4. Cannon Fodder: Every army needs foot soldiers. Though from the description of the world Covens would already be on top. Perhaps as muscle to keep the non coven people in line and handle mundane tasks?







                                                                                                                                                share|improve this answer












                                                                                                                                                share|improve this answer



                                                                                                                                                share|improve this answer










                                                                                                                                                answered Aug 30 at 19:18









                                                                                                                                                sean

                                                                                                                                                111




                                                                                                                                                111




















                                                                                                                                                    up vote
                                                                                                                                                    0
                                                                                                                                                    down vote













                                                                                                                                                    The Witch tax.



                                                                                                                                                    Witches are dangerous. Local governments fear nothing more than rogue covenants. Every girl born must be entered into the system. This is expensive. There is a tax collected from every girl's parents. This tax pays for school, social education and frequent visits from the "caretakers". This tax is not cheap.



                                                                                                                                                    If the parents can't pay the tax the child is drowned. Any attempt to hide a girl is punished by death.






                                                                                                                                                    share|improve this answer


























                                                                                                                                                      up vote
                                                                                                                                                      0
                                                                                                                                                      down vote













                                                                                                                                                      The Witch tax.



                                                                                                                                                      Witches are dangerous. Local governments fear nothing more than rogue covenants. Every girl born must be entered into the system. This is expensive. There is a tax collected from every girl's parents. This tax pays for school, social education and frequent visits from the "caretakers". This tax is not cheap.



                                                                                                                                                      If the parents can't pay the tax the child is drowned. Any attempt to hide a girl is punished by death.






                                                                                                                                                      share|improve this answer
























                                                                                                                                                        up vote
                                                                                                                                                        0
                                                                                                                                                        down vote










                                                                                                                                                        up vote
                                                                                                                                                        0
                                                                                                                                                        down vote









                                                                                                                                                        The Witch tax.



                                                                                                                                                        Witches are dangerous. Local governments fear nothing more than rogue covenants. Every girl born must be entered into the system. This is expensive. There is a tax collected from every girl's parents. This tax pays for school, social education and frequent visits from the "caretakers". This tax is not cheap.



                                                                                                                                                        If the parents can't pay the tax the child is drowned. Any attempt to hide a girl is punished by death.






                                                                                                                                                        share|improve this answer














                                                                                                                                                        The Witch tax.



                                                                                                                                                        Witches are dangerous. Local governments fear nothing more than rogue covenants. Every girl born must be entered into the system. This is expensive. There is a tax collected from every girl's parents. This tax pays for school, social education and frequent visits from the "caretakers". This tax is not cheap.



                                                                                                                                                        If the parents can't pay the tax the child is drowned. Any attempt to hide a girl is punished by death.







                                                                                                                                                        share|improve this answer














                                                                                                                                                        share|improve this answer



                                                                                                                                                        share|improve this answer








                                                                                                                                                        edited Aug 31 at 8:30









                                                                                                                                                        Ruadhan

                                                                                                                                                        2,564316




                                                                                                                                                        2,564316










                                                                                                                                                        answered Aug 30 at 18:57









                                                                                                                                                        Andrey

                                                                                                                                                        2,624726




                                                                                                                                                        2,624726




















                                                                                                                                                            up vote
                                                                                                                                                            0
                                                                                                                                                            down vote













                                                                                                                                                            Economics and Resources



                                                                                                                                                            Economics and the availability of resources has a huge impact on real world geopolitics and socials structures no matter which period you are looking at, you could use it in your world to explain at least in part the wanted behavior.



                                                                                                                                                            This point match with the "numerous ingredients" in your post. If resources aren't infinite in you world, there is probably a competition for resources between witches.



                                                                                                                                                            Witches would have a huge requirements of ingredients for several reasons :



                                                                                                                                                            • Casting spells

                                                                                                                                                            • Training in spells : how can you get better at magic without actual practice ?

                                                                                                                                                            • Increasing your own magic power : for example through regular consumption of elixir made from expensive components

                                                                                                                                                            • Building magical tools if they exist in your world (alchemy, wands, golems, magic jewelry, ...)

                                                                                                                                                            For all the reason listed above, the confirmed witches would already have a huge everyday cost in resources consumption and the training of new witches would also be extremely expensive. It would then be obvious it is better to focus the resources on a select few rather than share them along a lot of witches who will end up half-trained.



                                                                                                                                                            Chinese wuxia/xianxia novels use this mechanism a lot to drive competition between cultivators (people who train to become immortals like in Chinese myths)



                                                                                                                                                            If you combine this struggle for resources with other reasons that make it unacceptable to leave women untrained in magic, you get a mechanism to explain the imbalance in the male/female population.



                                                                                                                                                            This reason could be social stigma as suggested in another answer.



                                                                                                                                                            Or a stronger practical reason : every woman is born with magic whether trained or not and untrained, and if left unchecked accumulate mana over time, releasing surges of wild magic randomly (maybe only once a certain age threshold is passed).



                                                                                                                                                            These women would basically be walking time-bombs and witch training for young/teenage girls could easily be made compulsory (at least in civilized parts of the world, I'm not sure how medieval or modern your world is).



                                                                                                                                                            Combine the mandatory magic training with the limited resources and it could make giving birth to a female child a matter to plan ahead for most witches, making them use the sex choice spell to have male offspring most of the time as males would provide mundane workforce without the resource strain.






                                                                                                                                                            share|improve this answer


























                                                                                                                                                              up vote
                                                                                                                                                              0
                                                                                                                                                              down vote













                                                                                                                                                              Economics and Resources



                                                                                                                                                              Economics and the availability of resources has a huge impact on real world geopolitics and socials structures no matter which period you are looking at, you could use it in your world to explain at least in part the wanted behavior.



                                                                                                                                                              This point match with the "numerous ingredients" in your post. If resources aren't infinite in you world, there is probably a competition for resources between witches.



                                                                                                                                                              Witches would have a huge requirements of ingredients for several reasons :



                                                                                                                                                              • Casting spells

                                                                                                                                                              • Training in spells : how can you get better at magic without actual practice ?

                                                                                                                                                              • Increasing your own magic power : for example through regular consumption of elixir made from expensive components

                                                                                                                                                              • Building magical tools if they exist in your world (alchemy, wands, golems, magic jewelry, ...)

                                                                                                                                                              For all the reason listed above, the confirmed witches would already have a huge everyday cost in resources consumption and the training of new witches would also be extremely expensive. It would then be obvious it is better to focus the resources on a select few rather than share them along a lot of witches who will end up half-trained.



                                                                                                                                                              Chinese wuxia/xianxia novels use this mechanism a lot to drive competition between cultivators (people who train to become immortals like in Chinese myths)



                                                                                                                                                              If you combine this struggle for resources with other reasons that make it unacceptable to leave women untrained in magic, you get a mechanism to explain the imbalance in the male/female population.



                                                                                                                                                              This reason could be social stigma as suggested in another answer.



                                                                                                                                                              Or a stronger practical reason : every woman is born with magic whether trained or not and untrained, and if left unchecked accumulate mana over time, releasing surges of wild magic randomly (maybe only once a certain age threshold is passed).



                                                                                                                                                              These women would basically be walking time-bombs and witch training for young/teenage girls could easily be made compulsory (at least in civilized parts of the world, I'm not sure how medieval or modern your world is).



                                                                                                                                                              Combine the mandatory magic training with the limited resources and it could make giving birth to a female child a matter to plan ahead for most witches, making them use the sex choice spell to have male offspring most of the time as males would provide mundane workforce without the resource strain.






                                                                                                                                                              share|improve this answer
























                                                                                                                                                                up vote
                                                                                                                                                                0
                                                                                                                                                                down vote










                                                                                                                                                                up vote
                                                                                                                                                                0
                                                                                                                                                                down vote









                                                                                                                                                                Economics and Resources



                                                                                                                                                                Economics and the availability of resources has a huge impact on real world geopolitics and socials structures no matter which period you are looking at, you could use it in your world to explain at least in part the wanted behavior.



                                                                                                                                                                This point match with the "numerous ingredients" in your post. If resources aren't infinite in you world, there is probably a competition for resources between witches.



                                                                                                                                                                Witches would have a huge requirements of ingredients for several reasons :



                                                                                                                                                                • Casting spells

                                                                                                                                                                • Training in spells : how can you get better at magic without actual practice ?

                                                                                                                                                                • Increasing your own magic power : for example through regular consumption of elixir made from expensive components

                                                                                                                                                                • Building magical tools if they exist in your world (alchemy, wands, golems, magic jewelry, ...)

                                                                                                                                                                For all the reason listed above, the confirmed witches would already have a huge everyday cost in resources consumption and the training of new witches would also be extremely expensive. It would then be obvious it is better to focus the resources on a select few rather than share them along a lot of witches who will end up half-trained.



                                                                                                                                                                Chinese wuxia/xianxia novels use this mechanism a lot to drive competition between cultivators (people who train to become immortals like in Chinese myths)



                                                                                                                                                                If you combine this struggle for resources with other reasons that make it unacceptable to leave women untrained in magic, you get a mechanism to explain the imbalance in the male/female population.



                                                                                                                                                                This reason could be social stigma as suggested in another answer.



                                                                                                                                                                Or a stronger practical reason : every woman is born with magic whether trained or not and untrained, and if left unchecked accumulate mana over time, releasing surges of wild magic randomly (maybe only once a certain age threshold is passed).



                                                                                                                                                                These women would basically be walking time-bombs and witch training for young/teenage girls could easily be made compulsory (at least in civilized parts of the world, I'm not sure how medieval or modern your world is).



                                                                                                                                                                Combine the mandatory magic training with the limited resources and it could make giving birth to a female child a matter to plan ahead for most witches, making them use the sex choice spell to have male offspring most of the time as males would provide mundane workforce without the resource strain.






                                                                                                                                                                share|improve this answer














                                                                                                                                                                Economics and Resources



                                                                                                                                                                Economics and the availability of resources has a huge impact on real world geopolitics and socials structures no matter which period you are looking at, you could use it in your world to explain at least in part the wanted behavior.



                                                                                                                                                                This point match with the "numerous ingredients" in your post. If resources aren't infinite in you world, there is probably a competition for resources between witches.



                                                                                                                                                                Witches would have a huge requirements of ingredients for several reasons :



                                                                                                                                                                • Casting spells

                                                                                                                                                                • Training in spells : how can you get better at magic without actual practice ?

                                                                                                                                                                • Increasing your own magic power : for example through regular consumption of elixir made from expensive components

                                                                                                                                                                • Building magical tools if they exist in your world (alchemy, wands, golems, magic jewelry, ...)

                                                                                                                                                                For all the reason listed above, the confirmed witches would already have a huge everyday cost in resources consumption and the training of new witches would also be extremely expensive. It would then be obvious it is better to focus the resources on a select few rather than share them along a lot of witches who will end up half-trained.



                                                                                                                                                                Chinese wuxia/xianxia novels use this mechanism a lot to drive competition between cultivators (people who train to become immortals like in Chinese myths)



                                                                                                                                                                If you combine this struggle for resources with other reasons that make it unacceptable to leave women untrained in magic, you get a mechanism to explain the imbalance in the male/female population.



                                                                                                                                                                This reason could be social stigma as suggested in another answer.



                                                                                                                                                                Or a stronger practical reason : every woman is born with magic whether trained or not and untrained, and if left unchecked accumulate mana over time, releasing surges of wild magic randomly (maybe only once a certain age threshold is passed).



                                                                                                                                                                These women would basically be walking time-bombs and witch training for young/teenage girls could easily be made compulsory (at least in civilized parts of the world, I'm not sure how medieval or modern your world is).



                                                                                                                                                                Combine the mandatory magic training with the limited resources and it could make giving birth to a female child a matter to plan ahead for most witches, making them use the sex choice spell to have male offspring most of the time as males would provide mundane workforce without the resource strain.







                                                                                                                                                                share|improve this answer














                                                                                                                                                                share|improve this answer



                                                                                                                                                                share|improve this answer








                                                                                                                                                                edited Aug 31 at 11:57

























                                                                                                                                                                answered Aug 30 at 17:08









                                                                                                                                                                Alerand

                                                                                                                                                                12




                                                                                                                                                                12




















                                                                                                                                                                    up vote
                                                                                                                                                                    0
                                                                                                                                                                    down vote













                                                                                                                                                                    How about aging? If a witch is able to prolong her life for hundreds of years, each female offspring represents potential direct competition. If a witch has 2 daughters and in 20 years they are a match for her power what happens if one of them decides they really don't want to wait for mom to die off before grabbing that inheritance. What if they rebel against mommy and join a competing coven? Males, on the other hand, have no chance of affecting mom's power base so are much safer.



                                                                                                                                                                    Additionally, males though not able to directly use the magic still share half of their genes from their mother. Powerful witches could create alliances in an attempt to improve magic bloodlines by marrying off their male descendants to other powerful witches.






                                                                                                                                                                    share|improve this answer
























                                                                                                                                                                      up vote
                                                                                                                                                                      0
                                                                                                                                                                      down vote













                                                                                                                                                                      How about aging? If a witch is able to prolong her life for hundreds of years, each female offspring represents potential direct competition. If a witch has 2 daughters and in 20 years they are a match for her power what happens if one of them decides they really don't want to wait for mom to die off before grabbing that inheritance. What if they rebel against mommy and join a competing coven? Males, on the other hand, have no chance of affecting mom's power base so are much safer.



                                                                                                                                                                      Additionally, males though not able to directly use the magic still share half of their genes from their mother. Powerful witches could create alliances in an attempt to improve magic bloodlines by marrying off their male descendants to other powerful witches.






                                                                                                                                                                      share|improve this answer






















                                                                                                                                                                        up vote
                                                                                                                                                                        0
                                                                                                                                                                        down vote










                                                                                                                                                                        up vote
                                                                                                                                                                        0
                                                                                                                                                                        down vote









                                                                                                                                                                        How about aging? If a witch is able to prolong her life for hundreds of years, each female offspring represents potential direct competition. If a witch has 2 daughters and in 20 years they are a match for her power what happens if one of them decides they really don't want to wait for mom to die off before grabbing that inheritance. What if they rebel against mommy and join a competing coven? Males, on the other hand, have no chance of affecting mom's power base so are much safer.



                                                                                                                                                                        Additionally, males though not able to directly use the magic still share half of their genes from their mother. Powerful witches could create alliances in an attempt to improve magic bloodlines by marrying off their male descendants to other powerful witches.






                                                                                                                                                                        share|improve this answer












                                                                                                                                                                        How about aging? If a witch is able to prolong her life for hundreds of years, each female offspring represents potential direct competition. If a witch has 2 daughters and in 20 years they are a match for her power what happens if one of them decides they really don't want to wait for mom to die off before grabbing that inheritance. What if they rebel against mommy and join a competing coven? Males, on the other hand, have no chance of affecting mom's power base so are much safer.



                                                                                                                                                                        Additionally, males though not able to directly use the magic still share half of their genes from their mother. Powerful witches could create alliances in an attempt to improve magic bloodlines by marrying off their male descendants to other powerful witches.







                                                                                                                                                                        share|improve this answer












                                                                                                                                                                        share|improve this answer



                                                                                                                                                                        share|improve this answer










                                                                                                                                                                        answered Aug 31 at 17:10









                                                                                                                                                                        user3389672

                                                                                                                                                                        1063




                                                                                                                                                                        1063















                                                                                                                                                                            protected by James♦ Aug 31 at 19:04



                                                                                                                                                                            Thank you for your interest in this question.
                                                                                                                                                                            Because it has attracted low-quality or spam answers that had to be removed, posting an answer now requires 10 reputation on this site (the association bonus does not count).



                                                                                                                                                                            Would you like to answer one of these unanswered questions instead?


                                                                                                                                                                            Comments

                                                                                                                                                                            Popular posts from this blog

                                                                                                                                                                            Long meetings (6-7 hours a day): Being “babysat” by supervisor

                                                                                                                                                                            Is the Concept of Multiple Fantasy Races Scientifically Flawed? [closed]

                                                                                                                                                                            Confectionery